You are on page 1of 463
| \ = s* CO-ORDINATE ¥ he gees GEOMETRY Made Easy Published by : FIREWALL MEDIA (An Imprint of Laxmi Publications Pot. Ltd.) 118, Golden House, Daryaganj, ‘New Delhi-110002 Phone : 011-43 68 25 00 Fax : 011-48 53 25 28 info@laxmipublications.com ‘www laxmipublications.com Copyright © 2008 by Firewall Media (An Imprint of Laxmi Publications Pvt, Lid.) All rights reserved. No part ofthis publication may be reproduced, stored ina retrieval ‘stem, or transmitted in any form or by any means, electronic, mechanical, Photocopying, recording or otherwise ethout the prior written permiesion ofthe publisher. Price : Rs, 295.00 Only. First Bditio ‘OFFICES India Usa © Bangalore 080-26 61 15 61 Boston © Chennai 044-24 34 47 28 11, Leavitt Street, Hingham, ® Cochin 0484-299 70 04 ‘MA 02043, USA © Guwahati 0361-254 96 69, 251 98 61 © Hyderabad 040-24 65 23 33 © Jalandhar — 0181-222 1272 © Kolkata 033.2227 87 73, 22.27 6247 © Lucknow 0522-220 95 78 © Mumbai 032-24 91 54 15, 24 92 78.60 © Ranchi 0a51-290 77 64 Typesetied et: Shubham Composer, N. Dell Printed at: Pack Printere, Debi. ee Pe RP Be CONTENTS Chapter Cartesian System of Rectangular Co-orrinates Straight Lines—I Straight Lines—II Circle Parabola Ellipse Hyperbole Introduction to Three Dimensional Co-ordinate Geometry BREEBBe-@ craper 1 Cartesian System of Rectangular Co-ordinates 1.1, INTRODUCTION ‘We have already studied two-dimensional co-ordinate geometry in our earlier classes, ie., upto the high school, which is also known as “Euclidean Geometry’, as itis based upon the axioms laid by famous Greek Mathematician Euclid, around 300 B.C... The approach of Euclid was named ‘syn- ‘thetic approach to geometry’. ‘Co-ordinate Geometry is that branch of mathematics which deals withthe study of geometry by ‘means of algebra. A systematic study of geometry by the use of algebra was frst carried out by French Philosopher and Mathematician Rene Descartes (1596-1650) in his book ‘La Geometrie’ which was published in 1637. He incorporated the use of tools of algebra in studying geometry by establishing 1-1 correspondence between the points ina plane and the ordered pairs of real numbers called coordi nates of the points, The distinguishing characteristic of coordinate geometry is that it uses algebraic ‘methods and equations to gain information about geometric problems. The epproach of Descartes wes ‘named ‘analytic approach to geometry’. 1.2, RECTANGULAR OR CARTESIAN CO-ORDINATE SYSTEM Let two manually perpendicalar lines X‘OX and YOY intersected each other ata fixed point 0. Thea, @ The point O is called the orig (i) XOX is called the axis of x or x-axis. Mit) YOY is called the axis of or y-axis () XOX and Y’OY taken together are called ‘co-ordinate axis or rectangular co-ordinate axis (rectangolar because they ae a ight an- gles). Let a point P be drawn inthe plane. Draw per- pendiculars PM and PN on sans and y-axis respectively. 1 Co-ordinate Geometry Made Easy The real number associated with point M on -xeanis is called x co-ordinate (abscissa) of point P ‘and the teal number associated withthe point Non y-axis is called y co-ordinate (ordinate) of point P. ‘The ordered pair ¢r, y) is called the co-ordinates ‘of the point P. In stating te co-ordinates ef « point, the abscissa precedes ihe ordinate. The two are separated by 3 comma and are enclosed in 2 dracket ( ). Thus, 2 point P whose abscissa is “x” and ordinate is written as (x.y) or P 9). ‘The position of each point of plane is uniquely determined with reference to rectangular axes by ‘means of an ordered pair of real numbers, called ‘co-ordinates of the point. Conversely, let (t, 3) be any ordered pair of real numbers. Let M and N be the points om the axes XOX and Y'OY correspond ing tothe real numbers x and y respectively. Let the perpendiculars at M and N meet at P. The point Piis unique for a given ordered pai (x,y) of real numbers. Thus, we see that there is 1-1 correspond ‘ence between the points ina plane and ordered pairs of real musnbers. 1.3. SIGN CONVENTIONS FOR CO-ORDINATES (0 For Distances along x-axis : x-coordinate (or abscissa) of a point is positive if it is measured ‘along OX ice. to the right of O and is negative if itis measured along OX’ ie. to the left of O. (i For Distances along y-axis: y-coordinate (or ordinate) of a point is postive if i is measured along OY i.e, above the x-axis and is negative if itis messured along OY" .e., below the x-axis. Note: Any point A lying on a-axis has co-ordinates (x, 0) ke. Gy. Any point B lying on y-axis has co-ondinates 0, 9) i. (ily The covordinnes ofthe origin O ae (0, 0) ordinate is zero its abscissa is 2e10. 1.4, QUADRANTS ‘The coordinate aris X’OX and Y/OY divide the plane into four parts called quadrants, nurnbered I, I, ME and TV anticlockwise from OX as shown in te figure. y Cartesian System of Rectangular Co-ordinates 3 Both co-ordinates of a point in the first quadrant are positive as shown in the figure by (+, +). ‘The signs of the coordinates in the other quadrants are indicated as follows: (Quadrant ‘Sign of Co-ordinates ‘coordinate ‘yeoorinate (abscissa) (cette) 1 x>Otve y>0, tye Ho £6 0-ve yO, tye. More O-v yc0,-ve W__x>04e 1.5. DISTANCE FORMULA ‘In many problems, the distance between any two points, or the length ofthe line segment joining two points, is required which can be determined from the co-ordinates of the points. ‘The distance berween two points P (x,,y,) and Q (yy) is given by: PQ = (x -) +02 HF PQ = ¥(itference of abscissa)’ + (difference of ordinates)” ‘Tais result is knowm as ‘distance formula’ Remark : 1. The distance of a point P (x,y) from the origin © (0, 0) is given by op = y(x-0F +6-0) = Ye + 2, When the line PQ is parallel to the x-axis, the ordinates ofthe points P and Q willbe equa wes * PQ= Yla-a) HOn- w= Vie) =lanml- ‘3. When the line PQ is parallel to the yaxis, the abscissa of points P and Q will be equal. hen Xa t PQ = YH - a) +2 -m) =Vbr- =laoml 4 Coordinate Geometry Made Easy 1.6. SOME IMPORTANT POINTS 1, When three points are given and itis required to prove that they ae collinear, fe, hey ie om ‘the same line, then show that the sum of the distances berween two pairs of points is equal to the third pair of points. 2, When three points are given and iti required to prove tha they form : @ an isosceles triangle, show that two of its sides are equal. . (i) an equilateral rriangle, show that its all sides are equal, (ity a right-angled triangle, show thatthe sum ofthe squares of two sides is equal tothe square of the third side. ‘3. When four poins are given and itis required to prove that they Form (i) a square, show that all four sides are equal and diagonals are also equal. (ii) a rhombus, show that all four sides are equal and diagonals are not equal. Ait) rectangle, show tat opposite vides ere equal and diagonals are also equal. (>) a parallelogram, show that opposite sides are equal. ( aparateogram br nat a recent, how tt oposite sides are ul and agonal ee nt en 1.7, AREA OF A TRIANGLE Let ABC be a triangle with vertices A (x;, y,), B (xq, y3) and C (x, 93). Then, area of triangle ABC is given by = Area of AABC = 4 | x, 03-35) +1205 -¥) +5019) | amt nt mH yt The area of triangle ABC will come out tobe a positive quantity only when the vertices A, B, Care taken in anticlockwise direction Also, the area of a polygon can be obtained analysicaly, when coordinates of its vertices are given, ‘This can be done by dividing the polygon into disjoint triangles and then adding their areas. 1.7.1. Condition of Collineartty of Three Points ‘Three points A (xy, y,), B(x, ¥3) and C (r,, ya) lie on a Line if and only if the area of the triangle ABC is zero, Hence, thee points A (x, 9), B Uy, 93) and C (, 93) are collinear if and only if $1 Or-W +5 04-99) +4404 -99 | =O OR) FOS —ID FOL) =O and 1 and BY ‘ce. ifthe points A, B and C are collinear, then the area of triangle must be zero and vice-versa. o or, 2 Cartesian Systema of Rectangular Co-ordinates 6 1,8, SECTION FORMULAE Let A (i. ¥1) and B Gp, ,) be two distinct points, and any point P (x,y) lying on the line segment AB divides iin the ratio AP : PB. If the point P (x,y) ofthe line segment AB lis within the points A and B, then P is sid to divide AB internally, otherwise, the point P is said to divide AB externally If the point P (x, y) divides the line joining the points A (x,, y,) and B (rp, 9,) internally in the ratio . Thea, the co-ordinates of point Pare given by : mem. itm, mn men ‘The following diagram wilt belp to remember the section formula: aa Read Paw Beem) Note. 1 1 (3) the ido of the Hine joining A 9) am B i). Then, i vides AB in the rato 111 ‘The co-ordinates of mid-point P is given by Ath y uth 27 2 x= Keon ) Boe) Note. 2.1f the pol P¢,» ves the oe oinng the pos A, 3) and B,J) exteraly late ratio ‘m:n. Then, the coordinates of pine Pare given by xe MH yh ‘Note. 3, For problems in which it is required to find out the ratio when a given point divides the join of two unknown points, itis convenient to take the ratio a8: 1, for, inthis way two uskzowns ae reduced Ye one andthe formula becomes Bath y. Mth +1 unk ee ON AGG nD Pliny) Be va) 1.9. CENTROID AND INCENTRE OF A TRIANGLE - Let A Gj, 91), B ta, 2) and C Gry, ys) be the three vertices of a triangle ABC. Then, & Co-ordinate Geometry Made Easy () The co-ordinates of the centroid G (The point of con- currence of three medians ofthe triangle i called cemtroid) of the triangle ABC is given by Gay =o [HB IRAN) 3 3 Inthe igure AD, BE and CF are the medians of tiangle ABC and D, E and F are the mid-poins of sides BC, CA and AB respectively. Iv should be noted that centoid G divide each median internally in the ratio 2: 1. Biay) D Chey) Ate (ii) Let A, ¥,), B (Xp, Ya) and C (ts, y3) be the three vertices of a triangle ABC. Let AD, BE and (CF be te intemal bisector ofthe angles A, B and C respectively. ‘Then, the point of concurrence of three internal bisectors of the angles of a triangle i called its incentre Aen) ‘The co-ordinates of Incentre ofa triangle ABC is given by : Vey) = 1(Sieeates sitto), a+btc " atbee where BC = a, CA = band AB = ¢. 1.10. LOCUS AND ITS EQUATION 1.10.1. Definition ee a a Locus : When.a point Pmoves unde cern conditions then the pat traced out bythe point Ps caled the loeus ofthe point. Technically, a locus represents te ‘set of all points” which lies omit. The focus ofa variable point (ry is ealled a curve 1.10.2, Equation fo the Locus of a Point If (x, ») be a general point on the locus, then an equation involving x and y which is satisfied by each, ‘point onthe locus and such that each point satisfying the equation is on the locus is called the equation of the locus. In other words, the equation to the Jocus of a point is the relation which is satisfied by the co- ordinates of every point on the locus of the point. SEHMI Solved Examples FE Example 1. Find she distance between the following pair of points A, 3); B E32) CA 2,5): B-RD Solution, (9) The co-ordinates of given poimts are : AG, 5) and B (3,2). ++ By Distance Formula, we have. Cartesian System of Rectangular Co-ordinates 7 AB = f(-3-4)+Q-5" = (CCH = JOT = (é) The co-ordinates of given points are A (2, 5) and B (-3, 7) By Distance Formula, we have ABs (3-2 407-5) = (CSO = Bs = ‘Example 2. Find the distance between the following patr of poins : OA (aaj, 2atp ; B (at; 2at,) i) A (a.cos a asin a) ; B (a cos fh a sin f) Solution. () The co-ordinates of given points are A (a4, 2a) and B (at, 2ar,) "By Distance Formula, we have B= f(a? ai = fe [a=W ral +4 =a = 8 = 6F Gra) 40 =H = a(s-1) fla thy +4 (Gi) The co-ordinates of given points are : ‘A (@c0s cr a sin a) and B (a cos B, « sin). By Distance Formula, we have AB = (a.c0sB - acosa)’ +(asinB-asina)’ = {fe (cos p= cos olf =[o (sin B= sino)? = [a (cost B + cov? Peon cos) +a (om B + sin a Dein Pein] + (2ar, - 2ar,)* 8 Corondinate Geometry Made Easy a (cos? + sia® P) +a* (cos? a+ sin? @) - 2a" (cos acon ~ainasin) = {Fra (20 oosa-B) = {iP 2d eae B) (si? A + c08A = 1; 608 (AB) = (608 A cos B + sin A sin B) = 2 [l-cos(a-B)) ~00s28 = 2h? A 1-conA = 20? 4) P= ‘Example 3. (i) Find a point on the x-axis, which is equidistant from the points (7, 6) and (3, 4). (ti) Find the reletion that muss exist berween x and y so that (r, ) 4s equidistant from (6, ~ 1) and 3). (i) Find the reason that must exist benween x and y 50 that (x,y) ls equdstant from (- 1, ~ 1) and 42). (iv) Find a point on the y-axis, which is equidistant from the points (~ 5, ~ 2) and (3, 2). ‘Solution. (i) Let the co-ordinates of given points are : A (7, 6) and B (3, 4). Let P (x, 0) be the point on.x-axls which is equidistant from A and B. . Wehave PA = PB (Pay = By @-17 + 0-67 =@-37 +@-4" Baldr $49 4+ 36= 7 -6r+9 +16 = Mar + Ge = 25-85, ~ a= guuue u =8 2 _ eco eee in, Cartesian System of Rectangular Co-ordinates (i) Let the co-ordinates of given points AG, -1) and BQ, 3). Let P (x, y) be the point which is equidistant from the points A and B. Weave, PA=PB = (PAY = (PB)? = G-F ++I =@-9+Y-3 PEMA HELE a +d dr ty 49-6 = 1dr + 2y +37 =~ dr 6y +13 ~8 4 8-4 relation. (it) Let the co-ordinates of given points are : AGL, and B 4,2). ‘Let P (x,y) be the point which is equidistant from the points A and B. ‘We have, PA =PB = (Pay = (By = e-CDP+D-G DP =@-47 + 0-2" = G+ FOF &- 4 + -2F ms Pamela e bee Bet +P dy ts = tox + 61 = Sr + 3y-9=0 ‘Which i the required relation, (ty) Let the co-ordinates of given points are : A(-5, ~ 2) and BG, 2). Le (0, ) be the point on y-axis which is equidistant from A and. Wehne, PA = PB ay = 37 O+5% +0 +27 = 0-37 +0-2F B+ P+ sy t4a 9a yPady 44 4y429=-4y 413 16 yoo2 yuuugs 10 Co-ordinate Geometry Made Easy ‘The co-ordinates of required point is (0, - 2) Example 4. If he distance berween the points (a, ~ 2) and (5, 1) 5 units, find values of a. Solution. Let the co-ordinates of given points are A (a, - 2) and B (5, 1). It is given that, distance between A and B is 5 units. By Distance Formula, we have AB=5 = {6-0} +f1--2P =s5 = (S-a)+(42)" 5 {G= a) +9 = 5 (Squaring both sides) (S-a)? +9 =25 25 +a - Wa +9 = 25 @-1a+9=0 @-)@-9=0 anhs Example 5. )) Prove that the points (4, 4), (3,5) and (- 1, ~ I) are vertices of a right-angled triangle. (i) Show that the points (7, 10), triangle. Solution () Let the co-ordinates of the vertices of triangle ae given by : AG, 4, BG, Sand C1, - D. + By Distance Formula AB = (@-47 +(5-4)" = fn? +0)? = fiat = 2 (1-3) +13) = (64 C6? = 16486 = J and ace YE1-4 +C1-4 = JCS HCH = (BrB = LH * any = (2) (acy = (/33)' = 52 vuuvag 2, 5) and (3, ~ 4) are the vertices of an isosceles right-angled and (acy = (J50)" = 50. ‘Therefore, we observe that, sum of squares of two sides, AB and AC is equal o the square of the third side BC ie, (ecy’ = (aBy + (acy. Cartesian System of Rectangular Co-ordinates 11 ‘By converte of Pythagoras Theorem, triangle ABC is a right angled tringle and right angled aA. (iy Let the co-ordinates of the vertices of triangle are given by AQ, 10), B(-2, 5) and C 6, ~ 4 ‘By Distance formula : AB = Y(-2-77 +(5-107 = VCO +) = BFS = 108 Bc = [G42 4-4-5) = (6 +9) = (aR = /i08 and ac= {@-1 +-4-10" = (Cape? = fiers ~ (TE (aBy = (Y 708)" = 105 (BOF = (08) = 105 = (f203)' = ‘Therefore, we observe that sum of squares of two sides, AB and BC is equal to the square of the third side AC. and ac) (ACH = (AB + (BO? By converse of Pythagoras Theorem, triangle ABC right-angled triangle and right-angled aB. Also, the two sides AB= BC = /106 ‘The triangle ABC is an isosceles triangle. Example 6. (i) If (0, 0), (3, 0) and (x, y) are the vertices of an equilateral triangle. Find x and y. (Gi) The vertices of a triangle are (1, 23). (3. 0) and (~ 1. 0). Is the triangle equilateral, isosceles or seaene Solutio. () Lethe co-ardinates ofthe vertices of wiangle are given by: A. ©, 0), B (3, 0) and C (x, yp. By Distance Formula, we have, AB = f(3-0)' +(0-0F = (940 = J9=3 ay BC = y(r-3)' +(y-0) = fir-3 4 y? oe QD 12 Co-ordinate Goometry Made Easy and ac (@-0F += = Va = As itis given that, AABC isan equilateral triangle, We have, AB = BC = AC Lat AB = AC = ga (ray On squaring both sides, we get vtype ® Also, BC = AC = \o-3) +y? = (Fay ‘On squaring both sides, we get » @-3+harty = @-3 ae - Por +9—2 ( (di) Let the co-ordinates ofthe vertices of triangle are given by : A (1.2/3), BG, 0) and C (1, 0). By Distance Formula, we have, AB (e-1 +(0-2)3)" = =4 {er+-2y3) = varie Cartesian Sytem of Rectangular Coordinates 13 BC = f(-1-37 +(0-0F = [ar +0 = JT a4, ani ac= [cin +0-2)3) = (eae = (TT = ye 8. Therefore, AB = BC = AC ice, all the three sides of ABC are equal. Hence the triangle ABC isan equilateral triangle. Example 7. () An equilateral triangle has one vertex at (3,4) and another at (~ 2, 3). Find the co- ordinates ofthe third verte. (i If he two vertices of an equilateral triangle be (0,0) and (3, [3), find the third vertex. Solution, () Let the two vertices of an equilateral triangle be A (3, 4) and B (2,3). Let€ x,y) be the co-ordinates of the third vertex. + By Distance Formule, we have ap = (G42) 46-3) = (GP 0) = /BF1 = JH...) Be = (ara 6-3" @ and CA = J (x-37 +07-47° @) As it is given that ABC is an equilateral triangle. s AB = BC = CA (AB? = (BCP = (CA)? (AB) = (BC? (2) = [era 0-3" Woe tart 4+ P-6y 49 Pty tary = 13 6 . «Bcy’ = (CaP (x +2" +0-37 = Jox-3 40-4" PA +4 + POY +9 me P- Or + yt BY +16 4x-6y + 13 = - 6r-By + 25 Or +2y-12=0 Sr+y-6=0 ym 6-Sr a S) Putting this value of in equation (4), we have P+ (6 ~ Sx)" + 4x -6(6- 5x) = 13 Zeus u uuu 14 Coontinate Geometry Made Easy P+ 36 + 257-6 + 44-36 + 30K = 13 = 2? -2e-1=0 = a2 Fora quai xuson ar +x +0=0, aa-bt - po 2 ef = elt oO = en = Wd-s45f3 _ 7145/3 = an 2 2 ‘The co-ordinates of third vertex ae given by : te J3 7125/3) | (1-3 145/3 2 2 2 2 Gi) Let the two vertices of an equilateral triangle be A (0, 0) and B (3, J). Let € (, y) be the co-ordinates ofthe third vertex. 2+ ByDistance Formula, we have a= {0-07 + (3-0) = fay +(V3) = (953 ya ~-Q) artesian System of Reclangular Co-ordinates 18 ae = (r-37 +(9- 3) .@ and ac= {G-9 0-07 = {Hey 8) Asi is given that, ABC is an equilateral rangle. . AB = BC = AC = (AB)! = (BC) = (AC)? ‘ (apy! = (Ac? = (BY = (Fay = n-249 = Pte wo @ (Boy = (ac? = [ (e=3F+0- | (ry > P49 Get 43-2fFyar ty = ~6-2Y3y+12=0 = a+ fiy=6 = 97 oe ao) Putting this value of yin equation (4) we get =3:) - 2 (S3) <0 2 eG oy a BP +36 + 97 - 36x = 36 uuu 16 Co-ordinate Geometry Made Easy = £803. wen 20, $30) 6 3 _ 68 Ped = yous whee sodye GB 82 Bas OA = (AB) - (OB)? oD = OA = (2a) ~ (ay? = on = [4a -@ = {30 = J3.0 ‘The cocndinates of pict Ais (30,0) Cartesian System of Rectangular Co-ordinates: 24 Example 12, (i) Show shat the quadrilateral with vertices (3, 2), (0, 5) (~ 3, 2) and (0,3) in order isa square. (@) Show tha the four points (0, ~ 1), (6. 7). (~ 2. 3) and (8, 3) are the vertices of a rectangle, Solution. () Let the co-ordinates of given points are 9-1) evan AG, 2,B@, 5), C(-3, 2) and D (, - 1. By Distance Formula, we have B= (0-37 +6-2F = {C9 +OF = 79 = fie = 3/2 Be = {(-3-07 +@-57 = J(-3F +3) = (975 = fi = 3/2. a on cp = (043 +C1-2" BP +3) = J9ed vim ad DA={@-0)' +@+1) = {@ +O) = 999 = JB =A? AB = BC = CD = DA. ‘This implies that the quadrilateral ABCD is cither a rhombus or a square. ‘Diagonal AC = Y(-3~3)' + (2-2)' = V(- 6) +0 = 36 =6 Diagonal BD = fO—OF +C1-5F = fOr 6 = 3 =6 AC =BD “Ths al he four sides and diagonals ae equal, There- cra) fore, ABCD is a square, 0 (W) Let the co-ordinates of the points are A(@.-D,B(, 7), CC 2,3) and D @, By Distance Formula, we have AB= (6-07 +(7+1) = {OP +O = [558 = {105 = soa 22 Covontinate Geometry Made Easy cp = J@+2" +G-3" = ((lF +0 = J100 = 10. Be = [(-2-6 +0-9 = YEO +s = farts = (= 4/5 ap = (6-0 +641) = (OOF = (a = was wo - {6-9 +6-77 -{@rey = (FF = fH = 248 ac = (62-07 +40 = YER + = (4716 = J = 25 2 Wehave, AD = BC and AC = BD, This implies that quadrilateral ABCD is a parallelogram, Also, In AABD and in ABCD : (By? = (AD)? + DB? and (CD)? = (BC + (BDF ‘Thus, ABCD is rectangle. Example 13. Prove thatthe sum of the squares ofthe diagonals of a rectangle is equal the sum of he squares of ts sides. Solution. Let us suppose thatthe sides of rectangle be y and b units respectively. The co-ordinates ofthe vertices of rectangle i a sbown in the figure. 20.6) can Now, we have to show that : (Ac? + Dy" = (AB? + (BC) + (CD)? + (ADY’. LHS : > A = (Ac? + (BD? (a - OF + (-0F}] + [(a- OF + O- BF) (at +P +P + P= 2 +26 = 200 +B) ony Bao RES: = (AB? + (BCP + (CDF + (ADF = (a~ 9) + (0- OF + (a~a? + b-OF + O-aF +O = bF + 0-07 + b-0F aP+P ea +h amit we = LHS. =RHS. ' @ +e) Cartesian System of Rectangular Co-ordinates 23 Example 14. (i) Determine the centre of the circle on which the points (1, 7), (7 ue, What ts the radus ofthe circle ? (ii) Using Distance Formula, show that C (3, 3) isthe centre ofthe circle passing through the points A (6, 2), B (0, 4) and D (4, 6). Find the radius ofthe circle, Sotution () Let the points A (1, 7), B 7, ~ 1) and © G, 6) lies on a circle having centre at O (x, y) and radius We have, OA = OB = OC = r (radius). (OAy? = (0B) = (OC = 7... (1) (oay =? Gt o-F ar Pe Limetyt aay er P+ Pde My+ 5027 ..Q) (oBy =? G-P+ ote? Had Met y+ 1429=7 Payor say soe? -@) cory = 7 = @-8F + 9-97 =7 = BEGAN ty + 36-ye? > Pay 16e~12y +100 = 7 4) ‘From equations (2) and (3), we have Pobyh Be My +50 ax +y'— 1dr + 2y +50 = = 2ro My = He + 2y = 12x ~ 16y =0 and (8,6) vuugueUED > & = oS) From equations (3) and (4), we have xP +P Lae + 2y + 50 = + P= 1x By + 100 = olde + 2y = = 16r= 129 +50 = art May-50=0 = x4 17-25 =0 6 = x41(3s)-25-0 4 axa = te = 2s = 25x = 10-92 = 24 Co-ordinate Geometry Made Easy Putting this value of x in equation (S), we have yo Zayn3 ¢. The co-ordinates of centre is O (4, 3). ‘Alo, ais of cite, r= 0a = r= {OG 1F = (FHC = 418 = r= /B=5. i) The point C (3 3) is the centre of the circle passing through the points A (6, 2), B (0, 4) and DUO it cA =CB=CD By Distance Formula, we have : ca = (6-39 42-37 = rr =yori=yio 84 cB= (0-3 +4-3) = C3 +O = (941 = JO 764.8) and cp = {6-37 + 6-35 = (P+ = 5 = J it is clear that, CA = CB= CD =r (say) 0.2) Here, 1 = 10 is the radius of the circle. Example 15. (@) Let the opposite angular points af a square be (3, 4) and (1, - 1). Find the co- ondinates of the remaining angular points. (ii) The nwo opposite vertices ofa square are (2, 6) and (O, - 2). Find the co-ordinates of the other vertices, Solution (7) Let ABCD be the square with the vertices A (3, 4) and € (1, ~ 1) ‘Let (x, 9) be the co-ordinates of B * ‘AB = BC (Ce All four sides of a square are equal} (apy: = (Boy a &-F 4-47 6-1 + HP P+9-G +P + 6-H ae +12 ey HL ty ~ 6-8 +25 = 2+ 2y +2 4+ Wy-23 =0 ax = 23-10 BURG? Cartesian System of Rectangular Covondinates 25 (2) @ Now, in AABC, By Pythagoras theorem, we have (AB)? + (BC)* = (AC)? =~ 3 + G- 4 4-1 +O + 1% =G-18 +G + DF 4 9-6r ty + 16-8 ee Ld t y+ + dy =O + OF = WH Br- Gy +274 +S > cus) = 2e + 2y'-ar-6y-2 =0 = P4Pede-F-l20 uring the value of x in equation (3), we have ay-1=0 a BH OY ase ye ay 10 “ 529+ 1007 — 4607 + 67? + 1129-384 =0 Z 11g? ay + 15 20 = ay ay +5=0 = Qy-DQy-5=0 . ’ when ye *. From equation (2), we have : x = When yi s a-w(3) = From equation (2), we have :x = 1. The required vertices ofthe square are (ii) Let ABCD be the square with the vertices A (2, 6) and C (0, - 2). Letts.) be te coon of 8, ° cao AB = BC All four sides ofa square are equal) = (aby? = (acy? o > 6-2 + 9-6 = OF +g +2)? > P44 aes y+ 36-Ly=r ty ead ay = Ae + by = 36 = xt4y=9 d = ra9-4 OD Kew Been Now, In ABC, By Pythagoras theorem, we have (AB? + BOF = (ACF &-2F + - OF + e077 + + 2)? = O-2) + 2-67 Ped a +P 4 36-y trey tat aya sot 2x? + 2y? ae - By = 24 P4y-2r-4y-2=0 8) Putting the value of x in equation (3), we have (9-4) + 97-20-4y)-4y-12 = 0 sous > 81+ 16)? Ty + y= 18 + By-dy-12=0 = Ty? ~ 68y + 51 = 0 = a4 +30 = O- 0-3 +0 = yes re 9- 49) ‘when Lax=9-4() = 9-425 when yo 3ax=9-4G)99-12=-3, ‘The required vertices ofthe square are (5,1) and (~3, 3). Example 16.1 iste mio ofthe side BCof ring ABC, then, prove th: (aay + Uc? = 2 (AD) + (EF) ‘Sobution, Let the length of side BC is 2@ and is taken along x-axis. Ivis given that D is the mid-point of BC. Let y-axis bo the perpendicular to DC from D as shown in the figure. s. The co-ordinates of points B and C are (~ 2, 0) and (@, 0) respectively. Let Ana» Now, we have to show that (ABY? + (ACY = 2 (AD? + (CY) Aun em g cao Cartesian System of Rectangular Covordinates 27 os LHS. : (ABy + (AGF = & + 0)? + = 0) + e- a)? + OF aPad starlet ea toxrty arts aterd a1 +y +) o RES. 2[ADY + CY] = 2[ax- OF + Y-0) + (2-0) + 0-OF') a2 +e +a} | LHS. = RES. ‘Example 17. () Fad the area ofthe irangle whose vertices are (1, 6), (3, 0) and (- 3, ~ 7). (i) Find the area of the triangle whose vertices are (1, 2), (=2, 3) and (~3, ~ 4). (Ui) Find the area ofthe triangle whose vertices are (4,4), (3, ~ 2) and (- 3, 16). Solution. (2 Let the vertices of the triangle are A = (r,,y\) = (1, 6), B= (ty, 3) = (3, 0) and C= by, ¥9) = C3,-7. ‘Area of ABC = 1402-9) +205-9) +2501-99 | 11@ +1 +3C67-6+-3)6-| 2 =H iresemen) = 5 17-39-18] Ly soja 1-201 Ls0 = 2519, is (ii) Let the vertices of the triangle are A = (x), y) = (1, 2), B = Gy, y2) = (- 2, 3) and C= Gs ys) = 3-4) 2 ‘sea of AABC = 2 fm, 2-99 +2 05-¥) +45 001-99 | = FUB-CH+EDE4S-D+EH2-9| 1 = 7 1G 44-266-3-D] 1 1 L 74124 -ixn- its. pl TFiRe 3] m5 [22] = 5 x22 = A sq. um id Please Try Yourself. Ans. 27. sq, units 28 Coordinate Geometry Made Easy Solution. Let the vertices ofthe tiangle are A= (x, ¥;) = (kk 2) B= (i,9) = + 2,4 +2) and C = Gy, 99) = (E+ 3,0). 2. Theareaof ABC = 4 [5,099 +42 05-9) +4501-¥9 | -+ [kG +2-8 + (+2) R= +2) + R43) E-2-k-2) | oF [tk tk 4d 12] a) = basse ‘Thus, Area of ABC is independent of Example 19. (i) Find the aren of quadrilateral whose vertices are (2, 1), (6, 0), (5, ~ 2) and 3-0. (i) Find the area of quadiritateral whose vertices are (~ 4,5). (0, 7), (5. ~ 5) and (- 4, ~2). (ii) Find the area of quadrileierat ABCD whose vertices are A (1, 1), B (7, ~ 3, C (12, 2) and D2. Solution. (?) Let the vertices ofthe quadrilateral are A (2, 1), B (6, 0), € (5, ~2) and D(-3, - 1). Join AC. ‘Area of quadrilateral ABCD = Area of AABC + Area of AACD a) area of AABC = } [2042 +662-D+5U-0)| y Atea of = 4 |.x102-93) +2 03-9) #5 1-22 ca 4669451 ay ! wed 1 wd iaeuesy > Aren of AABC = 4.5 sq. units 1 @ Aa E160) ‘Area of SACD HD+) +S E1-1 FEA +2] Te AreaofA = $ 1x4 G2-39 +82 03-9) #3501799 IT Cartesian System of Rectangular Co-ordinates 29 1261) +562)-33)| -2-10-9/ +4 |-atj= baat > Area of AACD = 10.5 sq. units @) ‘From (1), (2) and (3), we have Area of Quadrilateral ABCD = 4.5 + 10.5 51g i (i) Let the vertices of the quadrilateral are A (- 4, 5), B (0, 7), C (5, ~ $) and D (- 4, - 2). tin AC. 2 Arnot ind ABCD « Aref AARC + Ara ACD 0 prot sabe = 2 | 6-90-65) #065-9 456-9 1102-99 + 03-9) +5 01-9) II 1 ene = 7 1-47 +9) 404562] 1 z 48-10] 1 ob ios) = bse => Area of ABC = 29 sq, units Aas fen @ Area of AACD = 2 | -4)E5-CD) + (492-9 +696-E5)] Gy Area of 8 = 4 [x1 2-99) +22 03-90 +45 01-99) II = hie Oeste t5E7-46545) | 12-35-40 => Area of AACD bcs = 31500 0 80 Co-ordinate Geometry Made Easy From equations (1), (2) and (3), we have ‘Area of quadrilateral ABCD = 29 + 31.5 = 60.5 sq. unis. i Please Try Yourself. ‘Ans. 132 sq. units. Example 20. For what value ofx, the area of the triangle formed by the points (5, - 1), t, 4) and (6,3) is 5.5 q. units Solution. Let the vertices of the triangle are A = (ry. yy) = (S, - 1, B = Gras.) = Or 4) and C = G9) = 6.3). & ‘Area of AAB $18 G2-¥) +0 05-7) 4501-99 | $1563 420-6) +661-4) | ISM +x6 +) +665] [524-0] +2 \4e-2s| But itis given that Area of SABC = 5.5 sq. units, 2 Wetave, f [ar-25 | =55 = Lar-25) 21 s Either x= 25 = U1 orde-25 == 11 = arm 11 + 28orax = -11 +25 = 4x = 3Gordx = 1d = sorx= 2 Example 21. (i) Show that the points (~ 1, - 1), (2, 3) and (8, 11) lie on a line. ii) Show shat the points A (b, ¢ + a), B (c, a + B) and C (a, b + c) are collinear. Solution. (() Let A (- 1, ~ 1), B (2. 3) and C (8, 11) be the given points. Area of ABC = > [4 02-99) +2203-9) +401 -») | [-1G-1D +201-C 9) +8C1-3)| [I-16 4201+ +864] Cartesian System of Rectangular Co-ordinates 31 | B+ 24-32| =0 Points A, B and C are collinear (i Let (b,c +), Bic, a +b) and C (a, b + ¢) are the given points, ‘Area of AABC = > | x; 2-93) +2 03-1) +501 -») | |bG@+b-b-9 +e) +c-c-a) tale +a-a-H)] |b@-0 +e@-a) +a(¢-B)| | babe + be ~ac + ac~ ab | Joj=o0 Points A, B and C are collinear. Example 22. (i) Find the value of x ifthe points (x, ~ 1), (5, 7) ond (8, 11) are colineor. () For what value of x will the points (2x, 2: + 2) (3, 2 + D, (2.x + 1) le ona line, Solution, (i) Let A (x, 1), B (5, 7) and C (8, 11) be the given points, 1 Area of A ABC = 5 |x 0:-¥) $2039) +3 01-¥9) | = bixo-myssar-emsser-a1 HE l-aesaiey+ece| 1 1 wh tet @-eed jes ti coer nl eb jeapfeet pe Peaxfret|a2xet| i z Now, the given pins are colnar i nl only ithe area of ABC = = 2jx#1/=0 = [rst] adore taom (LAA Qs, 2x +2),BG,2r + 1) andC (1, + I) be the given pois Area of ABC = 3 x, 03-99 +520s-9) #44099 | = acter + + 1+ 310+ )-@e43) + 12+ 2) +O [@+3Ex-D+1] [ae -ae-3 41] =} [2e-3r-2) Now, the given point ae collinear if and only i he area of AABC = 0 L flat-ar-2|=0 > 2?-3x-2| =0 = 2-3-2 =0 5 34 J9518 < _34f8 _ 32s +a 345 3-5 = — Fora quadrats equation, attic+c=0 “ ao Example 23. The co-ordinates of points P, Q. R and S are (~ 3, 5), (4, ~ 2). (®, 39) and (6, 3) respectively and the areas of triangles POR and QRS are inthe ratio 2 : 3. Find the value of p. Solution, Let P (- 3, 5), Q (4, ~2), R (P, 3p) and S (6, 3) be the given points. ‘Area of APQR = 3 1212-99) + 203-99) + 4301-99 | 1 3-2-3) + 4Gp-9) + pS-C2)] 163)@2-3p) +4Gp-5) + P6421 16 +9 + 12p-20+ 7p = Area of APQR = = | 28-14 | ao) Also, Area of AQRS = > | 1 Q2-¥9) +42.05-91) +4301 -¥») | Cartesian System of Rectangular Co-ordinates 33 = $14G7-3) +7 B-C} +6¢2-3)| = Flap 2+ 76 +2-12- 1ep| = Ep + 59-12-18 | 1 =1 ype De +2 ppl = 5 1 Do +20) L | sy lelilp ty | — ven of agns = |p +241 2 Now itis given that: Areaof PQR _ 2 Areaof AQRS = 3 1 4 )mp-4| ‘ 7 2? {By using equations (1) and @)) Flp+24 > mp-4| 2 peed | 3 rp-146 2 = pr ~*3 = Either o _2 2p -14 ptm 3 pie = 38-14) =2p +24) => 3 (28p-14) = -2(p + 24) = - = Example 24. The co-ordinates of A, B, Care (6, 3), (~ 3, 5) and (4, ~ 2) respectively and P isthe ‘point (sy) shove that APBC _|x+y- ‘ABC 7" Solution, Let A(6,3), B(- 3, 3) and C (4, ~ 2) are the given points and co-ordinates of point P be ey». 84 Coordinate Geometry Made Easy * Area of APBC = 35 (92-5) #42 (1-1) +4 (1 -¥4)] = Heae2-yrao-aea-c29) HE dC2-N 44-9) +4542) = F164 37-4 4y-204 70 1 1 Area of APOC = | e+ Ty= 14 |e [x ty-2| ) 1 Also, Area of AABC = > | 1 02 3s) + 32.0591) +.5.01-92) | = $ 166-6 +6962-9440-9] = F166 +9 +CDCH +469] 8 2 1 Area of BABC = 5/424 15-8 |= 3 [49| = . @ Dividing equation (1) by (2), we have : 1 apec _ ¥1#+¥-21 ase 7 2 = Wsty-2) _|sty-2| 2 7 Example 25. () If A (1, 2) and B (3, 8) be Iwo given points, find a point P such thet | PA | = | PB | and aPAB' = 10. (i) A, B are two points (3,4) and (5, ~ 2). Find a point P such that PA = PB and APAB = 10. Solution. () Let A (1, 2) and B 3, 8) be the given points, Let (r, ) be the co-ordinates of point P. 2 1PA |= | PB | on PA? = FBP @-1P + 9-2 = 4-3 +0 -8F PHL ty tangy 2 +96 ty + 64-1 24 De dy 4S wt + PG by +73 = Be ay +5 = =r 16) +73 e+ By-66 =0 x+3y-17=0 x3ye17 ° no, ‘Area of APAB = 10, ‘Area of triangle = 4 |x, (2-3) +42 04-94) +35 04-0 zuuuruuuys Cartesian System of Rectangular Corordinaies 35 a $ 116-9 +30-2+20-9| 10 Layo dy-6-6e1 10 |-6r+2y +2) =20 -6r+2y+2= 220 -Brtyti= x10 Either -3r+y=-11 -r+y=9 From equations (2) and (3), we have reSyed Also, from equations (2) and (5), we have rook ‘The co-ordinates of required point Pare cither (5, 4) or 1, 6). i) Lat A @, 4) and B (5, ~ 2) be the given pois. Let (x, be the co-ordinates of point P. [PA] =| PB) ~@ PA? = PB? @-3) + -4 = @- 5 +O +27 PH 9-Gr Hy + 16-B8y =H 25-1 HP Hd dy ac-ay=4 xe3ye = @ . ‘Area of APAB = 10 young @ 6 Zuusugs: = i [24-62 4302-9 450-9] =10 = [x4 +2)-6-3y + 5y-20| = 20 | 6x +2y-26| =20 = [3e+y-13| =10 = Brty- 135410 = Either atyeB ~~) ® u arty=3 From equations (2) and (3), we have redyn2 ‘And, from equation (2) and (4), we have re Toe cocdinates of required point Pare ctber (7,2) or (1, 0). 36 Co-ordinate Geometry Made Easy Example 26. Ifthe vertices ofa triangle have integral co-ordinates, prove that ihe triangle cannot be equilateral, Solution. Let A (,y:). B (tp )3) and C (ty, yy) be the vertices ofa triangle ABC. La Sod 1,2, 3, a imegers. 1 s Area of ABC = 5 1.4, 02-9) +4 05-9) 4504-99 | ‘= some rational number Let, if posible, AABC be an equilateral triangle sects sac = LE ie? 4 FB cap a (aay B . (a+ we integer) += an irrational number ‘which is a contradiction, because area ofa triangle is always 2 rational number. (ur supposition is wrong. ABC cannot be equilateral, Example 27, The vertices of a quadrilateral are (6, 3), (~ 3, 5), (4. ~ 2) and (x, 3x) and are denoted by A, B, Cand D respectively. Find the values ofx so that the area of triangle ABC is double ‘the area of triangle DBC. Solution. Let A (6, 3), B (- 3, 5), C (4, ~ 2) and D (x, 3x) be the vertices of the quadrilateral 10279) +203-) +4 O-W 1 Area of ABC = 165-2) +-92-3)+46-5] 165+2)+69-D+4-2] 49 ‘Area of AABC = [2415-8] =F) 49)-2 wo Area of ADBC = [4 02-99 +2.05-9) +45. 01-99 | 1@3)(62-3) +4Gr-5) +45-(2) | [649r4 124-20446+2)] Bie Bie Sie ee le Be ae Cartesian System of Rectangular Co-ordinates 37 [6+ 9+ 12r- 2047] =F lair p = Fes | eats = A axe 42e-1 2) But, itis given that: ‘Area of ABC = 2 (Area of ADBC) — [. By using equations (1) and (2)) 9 = Beraia-r 2-20 2-1) 9 = Be wiat 7 = jantp=2 1 = aetaat = Biter or z 1 = or tan = or 3 a = ond i. Example 28. Find the co-ordinates of the point which divides the line segment joining the points A, ~2) and B (9, 6) in the ratio 3:1. Solution. Let P (ey) be the required point which divides the line segment joining the points AG, ~2) and B @, 6) in the ratio 3:1 By Section Formula, we have NY 2 a a Bx941Ks | 745 a8 2 ae ae et Pkt a8 3x64 (1-2) 2 a exer lne?} +a ‘The required point is P (8, 4). Example 29. (i) Find the mid-point of the line joining (3, 5) and (- 7, - 3). (ty Find the co-ordinates ofthe midpoint ofthe line segment fobning the pots A (~ 2, - 5) and BG.-J. = sand DIAG, 5) and B (~ 7, - 3) be the co-ordinates of end-points of AB. and 38 Coordinate Geometry Made Easy Let P (, ) be the mid-point of AB. a : By Section Formula, we have ou 3-7-4 ‘The required point is P(- 2, 1) (ii) Let A 2, 5) and B G, ~ 1) be the co-ordinates of end points of AB. ‘Let P (x, y) be the mid-point of AB, 1 By Seen Foal, te ON -2+ tanta = 243 1 Kaa Pa” ba-1) 2 72 5+) ant 2 + Therap (!.~3) Example 30. () Find the co-ordinates ofthe point which divides the fin af PQ tn the ratio 2: 3 ‘internally and externally, the co-ordinates of P and Q being (2, ~ 1) and (- 3, 4) respectively. Also {find the co-ordinates of mid-point of PO. (i) Find a point on the line joining poines (0,4) and (2, 0 dividing the line segment (o) insernalty in reso 2 : 3. (©) externally in ratio 3 +2 Solution. () Let R (x, y) be the required point which divides the join of poins P (2, oes atniy nate By Sect Porm ae foro ga 2X C9432 606 | ao a ' 2 ° g 2x4+3x0) _ 8-3 _ 5 23 ‘The required point is R , 1) Let $ (x,y) be the required point which divides the jolt of points P (2, ~ 1) and Q (~ 3, 4) external inthe rio? 3. ‘By Section Formula, we have: . { mw oN is 7 sen ro) and y Cartesian System of Rectangular Co-ordinates 39 ot yo PAID 843 Ly 2 The required point is $ (12, ~ L1),. Let M (3) be the mid-point of PQ. By Section Formula, we have : ne : oN 4st - Fay wen ata. and y= itt 2 Merman or ‘é) (a) Lat P Gx, y) be the required point which divides the join of points A (0, 4) and B @, 0) {internally in the ratio 2: 3. By Section Formula, we have ga 2X2#3K0 | 440 at \ 2xO+3x4 _ O12 _ 12 2 = 2xOe3nd -2 —_____----- a ys 7 5S AG4) Pen 3.0) 2 Menaictacae (4.2) @) Let Q (x, y) be the required point which divides the joint of points A (0, 4) and B (2, 0) externally in the ratio 3: 2, By Section Formula, we have > 3x2-2x0 _ 6-0 _¢ 3-2 a2 3x0-2x4 _ 0-8 3-2 T ‘The required point is Q (6, - 8). Example 31. Find the co-ordinates of the points which divides the join of (1, 7) and (~5, 6) in she ratio (i) 1: 2 internally i) 3 : 2 externally, Solution. () Let P (x, ») be the required point which divides the join of A (1, 7) and R (- 5, 6) {internally in the ratio 1: By Section Formula, we have : we PxS)4201 542 3 * 142 -—____*+ _. Aga) ear 32.0) 40. Co-ordinate Geometry Made Easy we EET fy » eet There pis (-12). atin raya (Gi Let Q (x 9 be the required point which divides te join of A (1,7) and B (-5, 6) externally inthe ratio 3 : 2 + By Section Formula, we have 3x dy Co) rT) “The required point is (17,4). Example 32. Find the co-ordinates af the point which divides the join of : (i) (2, 3) and (5, ~ 3) in the ratio of 1 ; 2 internally. @) (2, 1) and (3, 5) in the ratio of 2 : 3 externally. Solution. Please try youre Ans. (9 @, 1) ©, -7). Example 33. The fine segment joining the points (3, ~ 1) and (6, 5) is trisected. Find the co- ordinaes of points of risection. Solution. Let A (3, ~ 1) and B (6,5) be the given point of ine segment AB. Let P and Q be the points of trsction i. (vides AB in three equal pats). or oT) ? @ Ba8) =PQ= =pps AP it Since, AP = PQ = QB=> 2AP = PB=> 5 = > P (x, ») divides AB internally in the ratio L: 2. By Section Formula, we have ow, 6)42%3 -646 4 1 2 1+? > —_+______. Ae=) Pay @ Bros, _1xS42KCD 5-2 3 L 12 3 30h Co-ordinates of P¢s, y) = (0,1) Now, PQ and QB are equal, = Qis the mid-point of PB. and y Cartesian System of Rectangular Co-ordinates 41 Co-ordinates of Q are given by : O+(-6) 145 { 2° 2 ) ($3) Example 34, 0} Find the ratio in which the x-acis divides the line segment joining the points 0, ~ 3) and (5,2). (1) In what ratio isthe line segment joining the pots (4,5) and (1,2) divided bythe xeas. Find also the co-ordinates of he point of eivision. Solutlon () Let the given points be A (7, - 3) and B (5,2) Let the line segment AB be divided by the x-ais at P (x, 0) inthe ratio k: 1, Here, we have to find the value of k For this, we use ony the ordinate (..,)-co-ordinate ofthe point P, which it 2er0.) £Q)+1C3) OS k+l Kea — Po.0) 86.2) 33) vent es AM nero 3:13 2 era) (io Let the given poims be A (4, 5) and B (1, 2). Let the line segment AB be divided by the x-axis at P (x, 0) in the ratio k: 1. Here, we have to find the value of k. For this, we use ordinate (-e., y-co-ordinate) ofthe point P, which is zero. EQ)+5()) O50 9 OS KM tit = BY coswts-0 fae Pee) Boa 5 w= -san=-8 = aae-$ svaxis divides AB in the ratio- 5:1 or-$:20r5 :2 (externally), Example 38, (i) Find the ratio in which the line segment joining poirus (3, ~ 6) and (- 6, 8) is cut by the y-avis (i) In what ratio is the line joining (4, 5) and (1, 2) is divided by the yoaxis. Find also the co- ordinates of points of division. Solution, () Let the given points be A (3, ~ 6) and B (- 6, 8). Let the line segment AB be divided by the y-axis at P (0, y) in the ratio. : 1. Here, we have to find the value of k. 42_Covordinate Geontetry Made Easy For this, we use abscissa (. -, reo-ordinate) of point P, which is zero. COW +16) _ 4 kal ‘yaxis vides AB in the ratio 3 + 1 or 1: 2 Gtr). i) Let the given points be A (4, 5) and B (1, 2). Let the line segment AB be divided by the y-axis, at P (0, y) in the ratio & Here, we have to find the value of k. For, this we use abscissa (ie, x-co-ordinate) of P, which is zero, kU) +14) sat a aan +4 oy oi - eA kas Pon = ke-6. Ban yas divides AB in the rio = 4: Lor 4 1 (exterally) ‘The co-ordinate is given by : AO) ye 2k+5 kL The co-ordinates of point P (0, ») = (0, 1), Example 36. In what ratio isthe line joining (2 ~ 3) and (5, 6) is divided by the () xaxis ; (i) y-axis, Also, find the points of division. Solution. Please try yourself, ‘As. (2) 1: 2 (internally) ; P 3, 0) (i 2: 5 (externally) ; P (0, - 9). Cartesian System of Rectangular Corordinates 48 ‘Example 37, Find he ratio in which the point P whose abscissa is 3 divides the join ofA (6, 5) and B (- 1, 4) and hence find the co-ordinates of poit P. Solution. Let point P (3, y divides the line segment joining the points A (6, $) and B (~ 1, 4) in the ratio &: 1 "By Seba Frm, we fave YOY s Ae)s6 ees gE, ET 7 eH hee tan Jen HAs = aks and y ket kel Sine, is ven tt, abc Le, 0-ortat) f point Ps 3, =k+6 Sak 6a3@41 kal 3 6=3¢ a > e633 > ao 3 ” red 3 ss Poin Py) dvides AB inte rato 8: Lor 3:4 Gtemaly 4k+5 2 Now, - ke 7" aT v a «Q)es = ye Jar a aes a > rT TT 4 4 Thecoonns oP 2.2). Example 38. () In what ratio is the line joining A (~ 1,1) and B (5, 2) divided by the tine x + y = 4. (i) Determine the raito in which the line 3x + y ~ 9 = 0 divides the segment joining the points (1, 3) and (2, 7). Solution. () Let the given points be A (~ 1, 1) and B (S, 7), Lette line x + y of A and B atthe point P (x, y) and inthe ratio k: 1, By Section Formule, we have : pe MOHED _ se i oN Test eed Kit kOe eet NOD Pinay Bin) kal aed divides the join 44 Coordinate Geometry Made Easy "cocina 9) (Now, this point P must lie onthe Line x + vu 1 = ued a kes The grenoedviesABin seo 412d, Now, a =I wyuores wiopepe ‘The point of division is (1, 3). (ii) Let the given points be A (1, 3) and B (2, 7). Let the line 3x + y ~ 9 = 0 divides the join of A and B ot P(x, y) inthe ratio & : 1 ‘By Section Formula, we have eS e9 pe ROVEL eH rd ket” kel Aaa) Poy) Ben (+13) _ 23 and ret 7 eT wes tess Couns pin Ps ie ® (54,2822) Now, the point P must lie onthe line 3x + y-9 = 0 Cartesian System of Rectangular Coordinates AB Ok+34TH+3-91R4)) _ 4 ¥el = 13k + 6-9-9 =0 = 4-3-0 kn 4 +The given line divides AB inthe ratio 2 <1 or 3: 4 internally) 2 Nox, = wet *T% 3 6, 0 aff) Sa 10 “Sa UE TG 4 44 Thk+3 a= “eT 3 4,, 3 4\+3 as B Gs 0 3 ay oT? 4 a4 10 33 The point of division is (42,22). romeriine (5.57) Example 38. (i) Show that she poinis (0, 3), (6, 0) and (4, 2) les on a straight line. (By using section formula), (i) Using section formu, show thatthe potnts (2, ~ 1), (2, 1) and (4, 5) are collinear. Solution. (f) Let the given points be A (0, 3), B (6, 0) and C (4, 1). ‘The three points will lie on a straight line if and only if any one of them divides the line segment Joining the other two points in a cerain finite ratio. vaca aneemers ease pear cea OR 4 = E40 ee eed A0.3 can 8@.0) and 1-088 al oe = 4 = 4+ Sb > Mk+sa6k = HH=4 ial 46 Co-ordinate Geomnctry Made Easy = k=2 and 1a oetiasska2 Fei We get the same value of k. Cis on the line AB Le., A, B, C are collinear. (i Let he given points be A (1, ~ 1) ; BQ, 1); and € 4, 5). he three points will lie on a straight line if and only if any one of them divides the ine segment ing the other wo in a certain finite rai Let C divides AB in the ratio k: 1 internally. By Section Formula, we have 4 = £1) at e410) ORTON an $ Re Ket = 2H a) Bey Rat = 4+) =H = akt4 we and > Sk4D Hk-1 Skt S ek-1 dk a6 = k *. We get the same value of k. Cis om the line AB Le, A, B, C ate collinear. Example 40. () Find the co-ordinates of the poinas on the line joining P (3, ~ 4) and Q (- 2, 5) ‘that is twice as far froma P as from Q. (i) Find a point on the line through A (~ 5, - 4) and B (2, 3) that is twice as far from A as from B. Solution, () Let the given points be P (3, ~ 4) and Q (- 2, 5). Let R (x,y) be the required point ice. it satisfies the given conditions. Now, there are two cases: Casel: Let R (x, ») divides AB internally in the ratio 2:1 By Scction Formula, we have Cartesian System of Rectangular Co-ordinates 47 ; $y = Roy) ar2.8) 2 262418) xo = 2+1(-4) _ 0-4 6 ad y= > 5 ro ‘The coordinates of pot R(x,» be (= 4 Case LetR (, be the external point of di By Section Formula, we have ‘The coordinates of point R (x, y) be (7, 14). (i) Let the given points be A (~ 5, ~ 4) and B (2, 3), Let C (x, y) be the required point ie, it satisfies the given condition. Now, there are two cases : Case I: Let C (r, y) divides AB internally in the ratio 2:1. “i wN +. By Section Formula, we have aey aa 2 2Q)41 2+1 4-501 AC) chy 4-5 ona 2G)+14) _ 6-4 _ , el 3 ++ The coordinates of point C (x, ») be Case: Let C (x,y) be the externa! point of division of AB ie., C divides AB extermally inthe ratio 2:1 By Section Formula, we have ~ 2@)-1C9) 21 2O-164 _ 6446 4g 2-1 1 - ‘The co-ordinates of point C (x, y) be (9, 10). Example 41. The co-ordinates of ro points A and B are (- 1. 4) and (5, 1) respectively. Find the co-ordinates ofthe point P which lies on extended line AB such that itis three times as far from B as Sroma, Solution. Let the co-ordinates of given points be A (~ 1, 4) and B (5, 1). Let (x,y) be the required point e., it satisfies the given condition. Here, C (x,y) divides AB externally inthe ratio 3 : 1. By Section Formula, we have 3-115) 3 nd y= a1 coon ornare (—a 2). Example 42. () The line segment joining (6, 3) to B (~ 1, = 4 is doubled in length by having half us length added 10 each end. Find the co-ordinates ofthe new ends. (i) The tine segment joining A (2, 3) and B (— 3, 5) is extended through each end by a length equal 1014s original tengi, Find the co-ordinates of the new ends. Solution. () Let the co-ordinates of given points be A (6, 3) and B (— 1, ~ 4). Let Py. yp) and Q (,y) be the co-ordinates of new ends. Now, According to given condition OS ap=lap jj? __} ___. 2 Pend Aa) Bea) 65-90) ‘he., A divides PB internally in the ratio 1: 2. ++ By Section Formula, we have : MEH) 5. HED420) 6 142 142 - i esteem = top a = a at Cartesian System of Rectangular Co-ordinates 49 Avignon: 3Q= 2 AB ‘he., B divides AQ internally in the ratio 2: 1 1 Seen Fora ene oS ON tH Pan) nea fa Ola 2(4)+10) 262) +10) ee MAT = p= +6 and 2y,+3 3 = <32m4+6 0 and -12 2,43 > 2-9 and 2y=~15 9 1s = ana ad => ‘The events of pit Qa. 39 8 (2.—!8). (i Lethe comrdinats of given pons are A @, ) and B (3,5) ‘Let P (x, ¥;) and Q (xp, ¥2) be the co-ordinates of new ends, —___+-____+>______+ Pen) A@3) 243.5) ate) Now, According to given condition = AB = AP =BQ 4.0, A is the mid-point of BP and B is the mid-point of AQ. When A is the mid-point of PB: = ats = md 3-45 = and Gay +5 md y= 1. ay ‘The co-ordinates of point P(x, 94) be (7, 1). ‘When B is the mid-point of AQ = > -3= Rt? 2 2 = -6 2x42 z fe EB 50 Co-ordinate Goomatry Made Easy = eb wd eT ‘The co-ordinates of point Q (xs, y_) be (-8, 7). Example 43, Prove that (4, - 1), (6,0), (7, 2) and (5, 1) are the vertices of a rhombus, Is i a square? Solution, Lette co-ordinates ofthe given point ae A (4, - 1), B6, 0), C (7,2) and D (5, Now, the co-ordinates of mid-point of AC is given by : (2.8) 62) 2727 ‘Also, the co-ordinates of mid-point of BD are piven by : CEP) - G4) AC and BD have te sme mito © (5) aving co-ordinates (5 = ABCD is a parallelogram, Now, by distance formula, we have : ap = fe s(0-Cay mac = 7-9 OF =e ory and - Oe = FFT = 5 and nViti = V5 ABCD is 2 parallelogram whose adjacent sides are equal. => ABCD isa rhombus. Also, AC = (7-47 +(2-(-) and Bb = 6-5" +-17 = er 2+? and = (wr+er = +a" and = ist Word and = = Vi8 = AC RD. Hence, all the four sides of ABCD are not equal. ABCD is not a square. Cartesian System of Reclangular Covardinates 51 Example 44, () Three consecutive ventces of a parailelogram ABCD are A (3, 0) ; B (S, 2): C2, 6), Find the fourth verter D. (li) Three consecutive vertices ofa parallelogram ABCD are A (4, ~ 11), B (5, 3) and C (2,15). Find the fourth vertex. Solutions. (?) Let the three consecutive vertices of the parallelogram ABCD are A (3, 0), B(S, 2) and C C2, 6). ow. ce, Let D (x, y) be the fourth vertex. nt aad Now, we know ut, the igonals of praio- ram bcc ach oer, ‘Mid-point of BD = mid-point of AC = cn v2) (34 §+0) 2° 2 2° 2 (ess222) Aaa B82 = FPer)-G4) +. Bguling he aesies on rds, we be a5 1 ys ot ma U2 25 2 2 2 = xesel wd y+2=6 - re amd ye4 ‘The co-ordinates of fourth vertex D (x, ») be 4, 4). (Gi) Let the three consecutive vertices of the parallelogram ABCD are A (4, - 11) ; B (5, 3) and c@. 15). aw ca.19) Let (, ») be the fourth vertex. Now, we know that the diagonals of a paral- Telogram bisects each other. ‘+ Mid-point of BD = mid-point of AC. v3) . (2 ais) 22 sac 86.3) ‘Equating the abscissas and ordinates, we have Coordinate Geometry Made Easy x+5 +3 a3 wd ee = 34526 wd yt3a4 = xel and yet + The co-ordinates of fourth vertex D (x, 9) be (- 4, 4). Example 45. if shree consecutive vertices of parallelogram are (~2, ~ 1), (1, O)and (4, 3). Find the fourth verte. Solution. Please try yourself. ‘Ans. (1,2) Example 46. Three consecutive vertices of a rhombus are (5, 3), (2, 7) and (~ 2, 4). Find the fourth vertex. Solution. Please try yourself. Hint : [Rhombus isa parallelogram wit ‘Ans. (1,0). Example 47. (i) The points (- 1, 6) and (5, 4) are opposite vertices of @ square. Find the co- ordinates of the remaining two vetces. (id If m0 adjacent vertices of a parallelogram are (3, 2) $&—_______¢4) ‘and (~ 1, 0) ard the diggonats intersects at (2, ~ 5}. Find the other two vertices ofthe parallelogram. Solution. () Let A (~ 1, 6) and C (5, 4) be the given opposite vertices of the square ABCD. Let the Diagonals AC and BD meets at point O. @ ‘Co-ordinates of point O are given by : m145 644) _ (4 10) eS +) G3) Let (x, ) be the co-ordinates of any of the remaining two ACA. vertices. ‘Then, distance of point (x,y) from O = Distance of point A from O. [+ Centre of square is equidistant from the four vertices) all sides equal.) 2, 3). 4+ By Distance formula, we have : Ve-2" +0-57 = = (-2F 0-97 =YE3F +07 (-1-27 +6-57 Cartesian System of Rectangular Co-ordinates 63 = (x-2/ 4-5) = 951 = 10 = 6-27 +y-57 = 10 s+ (2) [On squaring both sides] V(x-5) +4)" Vern 40-97 = fix-57 40-47 [On squaring both sides] +I + 9-8 = 4-57 + 0-47 PALE De Hy + 36- 12y = x7 + 25-10 +»? + 16- By PP 4 Deo 12y 437 a byte TO By 41 Leap as ae-yed yoked @ Substituting this value of yin (1), we have @-2) + Gr-1-5? = 10 vauuuny = Ped-de + Gr-67 = 10 Pade +9 436-36 = 10 = lox? 40r + 300 = Paar +3=0 = @-3)@-D=0 = xe3orr=1 2 From 2); yo3()-1=3-1=2 when From 2), y=3G)-1=9-1=8 The co-ordinates of other two vertices of square are (1, 2) and (3, 8) i) Let the given adjacent vertices of parallelogram are A (, 2) and B (~ 1, 0). Let the diagonals ‘AC and BD intersects at point O (2, - 5). Let C (4, y,) and D (xy, 3) be the remaining vertices of the parallelogram. [Now, we know that, the diagonals of a parallelogram bisects each other. 54 Co-ordinate Geometry Made Easy 0 is the mid-point of AC as well as that of BD. ‘When © (2, 5) is the mid-point of AC. Sem 2m 2 bin) Cour) = and -10=2 +3, = andy =-12 + Co-ordinates of point € ¢x,,y;) be (1, ~ 12). and ‘When O (2, ~ 5) isthe mid-point of BD. = 148 and-5 = 2 2 2 Aaa (1.0) = 1+ xamt-10=y = Sandy, = - 10. Corondinates of point D (x, 33) be (5, ~ 10) Example 48. The centre of a circie is O (- 2, 5) and one end of a diameter is P (3, ~ 7). Find the co-ordinates of other end. Solution. Let one end of the diameter is P (3, - 7). Let the other end of the diameter is Q (x, »). Since, O (- 2, 5) is the centre of the circle, 0 is the mid-poia of the diameter PQ. = Tandy = 17 Past The co-ordinates ofthe other end of diameter Q (x. ») be (7, 17). Example 49. Find the area of the triangle formed by the ‘mid. points of ides of te triangle whose vertices are (2,1). Aen 2, 3) and (4, ~ 3). Solution, Let the given vertices of triangle are A (2. 1), B(-2, 3)and C4, = 3). Let D, E and F be the mid-points of the sides BC, CA and AB respectively. 5 e Co-ordinates of point D are ie. 0). ay ° cum Cartesian System of Rectangular Co-ordinates 86 Corn of pin Bae (22 Cort fp a (2 ‘Area of wingle DE i given by : B25 Ln +H0r-9 +0199 | = } [11-2) +32-0) +00 +1)| 1 Bs wn = Ep -se6 40] = 3 sq unis, Example 50, The vertices of a triangle ABC are A (-7, 8). B (S, 2) and C (11, 0). If. E. Fare the mid-points of the sides BC, CA and AB respectively. Show thei : AABC = 44DEF. Solution. Let the given vertices of triangle are A (~ 7, 8), B(S, 2) : and C (11, 0) Let D, E and F be the mid-points of the sides BC, CA wane and AB respectively. + Co-ordinates of point D are (8 20) 6 (fs 2 ou) te. (F ) an coors opin Ea (S=M, Oe ie. ($2) te Co-ordinates of point Fare (; 2,4). on ° em,0) -743 8 2 Now, Area of ADEF is given by 1 ans [402-99 +2.05-¥) +5 01-9) | =£1sa-9426-y+e00-9) Lien s2@-1691 86 Co-ordinate Geometry Made Easy ‘Also, Area of SABC is given by [84843 |= 2 ca unis, 14 02-9) +205-¥) +45 01-99) | -72-0) + 50-8 +11 G-2)| = 14-40 +66] = 1 | 12] =60q uals, ‘ sane « (2) ~on¢ sie (2, ~ 1). Find the co-ordinates of its vertices. Solution. Let A (x, 9)s B (i, ») and C (2, 5) be the vertices of ABC. Let D (1, 2), (0, ~ 1) and F (2, ~ 1) be the co-ordinates of mid-points of sides BC, CA and AB respectively. ‘Since, D is the mid-point of BC We have, Beth tang BAD 2 2 = Atye2 and Rtas ‘Also, E is the mid-point of AC. We have +n + ¥ Go = Oand AS = at and at ‘Similarly, F is the mid-poiat of AB *. Weave : ath 2 atm 2 = Zand 5% -- AABC = 4 (ADEF) Example 51. [fhe co-ordinates ofthe mid points ofthe sides ofa triangle are (1, 2), (0, ~ Dand Aba) Fe-r 5-1) a) -@) Pon ou Clad + @) o +) Cartesian System of Rectangular Co-ordinates 57 and Ntyy=-2 oH ® ‘Adding equations (1), (3) and (5), we get tnt tyta ty slt0rd = 2x + Oey +245 =6 = atntyss - Adding equations (2), (4) and (6), we get HAWN ty tN twa 4-2-2 = By +22 +2520 > AtkTWED ® From equation (7), we have nt? lat = y and from equation (8), we have y+4=0 lity 4) = nant Covordinates of point A are (1, 4). Also, ytytys3 > mt0=3 [ey tx <0) = n=3 nd ntnty=0 = wtED=0 (ey tys=-21 = ye? 2 Co-ordinates of point B are (3,2) Further, atu tya3 = Soya tray tay) = wyeel and ntnty=0 > -2+y3=0 mtd = yn? Co-ordinates of point C are (~ 1, 2) ‘Thus, the three vertices of AABC are A (1, ~ 4), B (3, 2), and C (~ 1, 2) Example 52. The vertices ofa triangle are (2, 2), (0. 6), and (8, 10). Find the co-ordinates ofthe trisection point of each median which is nearer 10 the opposite side 58 Co-ordinate Geometry Made Easy aaa 808 Dae 18.1) Solution. Let A (2, 2), B (0, 6), apd C (8, 10) be the given vertices of ABC. Lat AD, BE and CF be the medians of ABC. sD, End F be the mid-points of sides BC, CA and AB respectively. counts ofp Dae (228,59) ie, (8.59) ses 8) Coors ti Ee ( (2.3) 0.8.6 counties Fae ( ie. Ba)iecae. Let O, be the tisection point of the median AD which is nearer to the opposite side BC. ©, divides AD in the ratio 1 : 2 internally. By Section Formula, we have : (ae wa 12 Tee - es 248) =(2 8) -(2 6) aS 373) 43" Let O, be the tiseetion point of the median BE which is nearer to the opposite side AC. (0, divides BE in the ratio 1 : 2 internally. ++ By Section Formula, we have : oy = {MO#21) 116) + 216) ae 142 Cartesian System of Rectangular Co-o (32.522) -(2.4) -(2.) Let Q; be the trisection point of the median CF which is ncarer tothe opposite side AB. (©, divides CF in the ratio 1 : 2 internally By Section Formula, we have 18)+2(1) 100)+2(4) o- GO =(# 1028) = (# 8) @ 6) 3° 3 373 37 Ts, sO, 0a 0, we inet, dy bn tee settee. (2.6) Example $3. (i) Two vertices ofa triangle are (3, - 5) and (~ 7, 4). Ifthe centroid is (2, ~ 1). Find the third verve, (0 The co-ordinates ofthe centroid of ariangle are (3,2) and ew ofits vertices are (23. ~ 1) and (213, 3). Find he co-ordinates of he third vere Solution. (?) The given vertices of the AABC are B (3, ~ S) and C (~7, 4), Let A éx,y) be the co- ordinates of the third verte, Co-ordinates of centroid G of AABC are given by Alex) (atnts, 3 fry c(7.4) - es 60 Co-ordinate Geometry Made Easy and yrls-3 = x=10 and yer? ‘The co-ordinates of third vertex is (10, - 2). (id The given verices of AABC are B (2Y3,-1) and C (2/3.3). Let A Gx, 3) be the co- ordinates ofthe third vertex. Aen Co-ordinates of the centroid G of AABC are given by = (atgts ntytn) 3S sep calis Bot i is given that, co-ordinates of centroid be (3.2) . aii 5 3 yea ad ar = 44/3 = 3 aod yran ~ x3 and yea The eocrdinates of itd venen are (~ 3,2). Example $4, Find he centroid ofthe triangle with vertices (1, 7) (= 3, ~ &) and (~ 6, 4). Solution, Let the given vertices ofthe triangle ABC are A (1, 7), B(- 3, - 4) and C (- 6, 4). Let G (x, y) be the centroid of AABC. oyy= (S2GTB eet) Cartesion System of Rectongular Covordinates 61 aa.) c¢6,4) Example $5. (f) Find the Incentre of ihe AABC with vertices A (4, - 2), B (~ 2, 4) and C (S, 5). (ii) Find the incentre ofthe triangle whose vertices are (~ 36, 7), (20, 7) and (0, - 8). Solution, (?) Let A (4, - 2), B (- 2, 4) and C (5, 5) are the given vertices of SABC. Let I (, y) be the co-ordinates of AABC. Bt +e aagy — MED HOD Fe abse ah ose wo By Distance Formula, we have : a= ac} = (6-2 46-47 e . = (OP +1 = (BF = J = 5/2 vejcale (ease tan cas) = (OOF = JB = f50 = 5/2 = AB] = {2-9 (4427 = (COO = (F5 = JH ~ 62 Wehave yn tht +e, V2) + 6/2) 29+ (6/2), atbec «S245 f2+6f2— 62_ Co-ordinate Geometry Made Easy _ 22 -10/2 +30f2 ~ 16/2 soz *te2 ~2 Bit bys soe _ (512) (2) + (592) 4) + (6Y2) 09) aebee 32432 +6]2 10/3 +207 + 30/7 16/2 ? 98.3 ist “3 4 The co-ordines of inseatre are (3.3), (Let A (- 36, 7), BQO, 7) and C (0, ~ 8) ae the piven vertices of ABC. Let Gy y) be the co-ordinates of AABC. Aven pe Githy toy arbre ath +0, arbte By Distance Formula, we have 5 te . [Bc | = (0-20) +(-8-7)° = (C207 +157 = (TOOTS = JOS = 25, 27 . CA | = YR 36-0F +058" = (G5 HO = (TOT DS = JTS = 39. = | AB | = (20436 += 7F = [GHP +0 = 56. We bave e co.) 25(~36) +39 (20) + 56 (0) ax, + bx, +0 | a+hee 25439456 artesian System of Rectangular Co-ordinates = 7900+ 78040 _ -120 120 120 aol and y= Mtb +o, _ 25(04390) 456-8) arte 25+ 39+ 36 Example 56, [fthe co-ordinates ofthe mid-poirus of the sides ofa triangle are (1, 1), (2, 3) and (8, 4). Find its centroid and in-centre. Solution. Let A (x, ¥:), B (fy J) and C (4, y,) be the three vertices of AABC, Let D (1, 1), E @, ~ 3) and F G, 4) be the mid-points of sides BC, CA and AB respectively. Since, D (1, 1) is the mid-point of BC Aten) * ap ~ (S32 abs) 2 2 b = fhe Fey cay and y = hte? = @) wd yy ty 2 @ Also, E (2, ~ 3) is the mid-point of AC Btn) pan Clty Ho) : == (825,022) e @-9= (Sta, 48 = Aen A =2 WAH and Beans = aban 0) and wtye-6 « Further, F G, 4) is the mid-point of AB. ~ (2) = age = Sand Lh =4 64 Co-ordinate Geometry Made Easy ond Atyn8 Now, adding equations (1), (3) and (5), we have enty tytn tynlt+4+6 © © = Dy + Day + Dey = 12 > nents o Adding equations (2), (4) and (6), we have : Yat Mt AY tH HM 2-648 = +22 += 4 > Wty t ye? xO} ‘From equation (7), we have : ytmtys = +2 [omy tx = 21 = x=4 and Ya td2 +9 By using (89) wre = a +The co-ordinates of point A (1, ¥1) be (+, 0) From equation (7), we have Ate tH 6 = mt4=6 = % ita =) and Wty ty =? [> By using (8)) at CO=2 = ya246 = ys8 ‘The co-ordinates of point B (c, )2) be (2,8). Again, from equation (7), we have : = xy tx ty 26 boxy tx = 6) = ayt6= = 4% and tye t= [> By using @)) = yt 8=2 = ya-6 % The co-ordinates of point C (ry, ys) be 0, ~ 6). Cartesian System of Rectangular Co-ordinates 65 Kow, an B= ft-0) +(0-6) = (@F + Ore? = Y16436 = (52 = 2/13 b=ac= 4-2) + 0-8 = farecay? = VERE = Je = NTT and cane {2-0 +840" = [OF + Uay = YFF196 = 20 = 102 ‘The co-ordinates of incentre of AABC are given by Bit bey +e, a + bys +O Grbac athe jeer (2473) @) + (2/77) (- 6) + (10.2) (0) 2fis +2J7 + 10f2 . 2Yis 42/17 +102 . 4/B+0+40J2 16/73-12/i7 +0 2Y13 +217 + 10,2 * 2/13 + 2/17 + 10 J = 2fii+23 8/7367 Tae fiesia Tas J+s]i Which ae the required co-ordinates of incenie of ABC. ‘Now, the co-ordinates of centroid of AABC is given by = + ity. [eee estas 3 3 = 44240 0484-6) 3 CUS 6 8-6) _(,2 - 6-54)=@5) Example 57.17(3, 0), (3,6) ana 1, 6) are ipo of the sides of iat, then fr she Incerare and centroid of atlangie. Solution, Please try yourself. Hint: Procesd as in Example $6. 66 Co-ordinate Geometry Made Easy how const (2.4) tt .2) ‘Example 58. Find the equation of the locus of a point which moves so that : (atten 15 era nae (0 aay nt fom ye ‘Solution () Let P (x, ) be any point on the required locus. y ‘ . 3 Pil 2 las int * io y Draw PQ L sans, so that, PQ = 2.5 + y= 2.5 isthe required equation of ies. i Let P (x ») be any point on the required locus. y 4 y b Now, Point P (x,y) lies either on perpendicular line /, or on line /s, which are ata distance of 5 units from y-axis. | Cartesian System of Rectangular Co-ordinates 67 For any poins P(x, y), we have x= 44, which is the roquired equation of locus. Example 59. ()) Which of the following points lie on the locus given by the equation. 2+ OP -5e=8 (0) 2, 1) and (0) (0,2) (@) Find the equation ofthe tocus ofa potne which moves so that is diiance from the x-axis ts five times its distance fromthe y-aui. Solution. (3) The given equation ofthe locus is: 2d +2 -3e = 8 @) (@) The co-ordinates of given poiat i (~2, 1). From (1), we have : 2627 +207 -3-2 98 = 2) +20) +6=8 = ee2t > 8 = 0, which is not rue. ‘The point (- 2, 1) does not lie on the locus given by equation (1). (@) The covordinates of given point is (0,2). From (1), we have 20% +2@P-3()=8 048-058 8 = 8, whieh is tue, ‘The point (0, 2) lies on the locus given by equation (1). (0 Let @, ) be any point om the Locus. u on 4 Draw PQ and PR perpendiculars on x-axis and y-axis respectively. Now, its given that 7 [PQ] =5| PRI = lyl=Siz] = y= 257 ‘which isthe required equation of locus. Example 60. (i) The sum of the squares of the dissances of a moving point from nwo fleed poinis (@, 0) and (- a, 0) is equal to a constant quantity 2c Find the equation of ts locus. (i) Find the equation to the locis of a point equidistant from the points A (1, 3) and B (~ 2, 1). Solution. (() Let P (x, y) be any position ofthe moving point. Let A (a, 0) and B (~a, 0) be the given points. Then, according to the given condition, we have : PA? + PB? = 22 (> By Using Distance Formula] 88 _Coundinate Geometry Made Easy = a + 9-08 + eC ayr +y-0F = 22 = a +P tw tartare > FA@-tar see ea + tart y'=22 = at 42? 2d wad = Payleaad = Beysc-e “which is the required equation of locus. i) Let P (, ») be any point on the locus. Let A (1, 3) and B (- 2, 1) be the given points. Then, according to given condition, we have = PA = PB = PA? = PBY By Distance Formula] = @- IP + Q-3P = @-62F + 0-17 = o-Ps 9-3 so +2 + 1F SPIE HOFer eee ty EIA 2y = ater +10 = =2y +67 +5 = ~6r = 4y-S = be + ay =5 ‘which is the required equation of locus. Example 61. (i) Find the equation of the locus of a point which moves so that its distance from 3, 2) is twice tis distance from (1, 1). i) Describe the locus of the point P x, y) satisfying the equation (x ~ 2)* + (y - 33° = 25. Solution. () Let P(x, ) be any point on the Focus. Let A (3 2) an B (1, 1) be the given pois. ‘Thea, according wo the given condition, we have PA = 2PB = PA? = 4B? By Distance Formula} = -3P +2 = 4foe- PF + O- PHO tha dy df 1-2 +P 41-29) = By 6e-4y 413 = 4 [e+ -2e-29 +2) = 24 Pp ~fx-4y +13 = ae + ay? Br By +B = ar + 3)?-2r-dy-5 =0 ‘which isthe required equation of locus, (ii) The given equation of locus is = e- 27 + y- 3)? = 28 = Y@-27 +0-37 = Let P (x, y) be any point on the locus, Also, from (1) Point A (2, 3) lies on the given equation of locus. . Wehave, PA“ S > Point P(x, y) moves so that its distance from the point A (2, 3) is always 5. + The locus of P(x, y) isa circle with centre (2, 3) and radius 5. ‘Example 62. Find the equation of she locus of a point such that she sum of lis distances from 1wo fazed points is constant and equal to 2c. Solution. Let ewo fixed points be A (- a, 0) and B (a, 0) lies on x-axis having origin O as their ‘mid-point. Let P (4, y) be any point on the locus. Then, according to given condition, we have [PA | +| PBI = 2c [By Distance Formula] 3 fle- Cay +y-0F + tra) +07 = 2¢ 2 (ray ty tyle-a)' ty? = 20 = (rsa) 4y? =20-f@-ap+y (On squaring both sides, we have et aP sy a4e tora? ty-ae(e-ay ey = Pee branded Pal tar-de {E-a ty = 2av = 4c - 2a ~4e J (xa)? +9’ = de qle-a) +y? = 4 =dar = {enor ey (On squaring both sides again, we have teat ey =(o-44) = Padme tye + SF par 70 Coordinate Goometry Made Easy = (-S]F er ee-e = on@ ay peed # y 7 2. = Segtgel [Dividing both sides by (2 =) whieh isthe required equation of the locus. posal 6A rade ee 2 ales bce repre ine, de oa fm . (i) Fad the locus ofa point suck that the surn ofits distances from the potnts (0,2) and (0, -2)18 6. Solution. () Let AB be the line of length 22 meeting x-axis at A (a, 0) and yuaxis at B 0, b) Let OA = @ and OB = b. Since, OAB is right angled triangle. By Pythagoras theorem, we have OA? + OB? = AB’ = (a-0F + @-0) + (0-07 + (-0F = QF = @+R=aP @ Now. the co-ordinates of mid-point of AB is given by : Cartesion System of Rectongular Co-ordinates 7A ) a= 2randd = 25. Putting these values ofa and bin (1), we get: => (2x) + Qy? = 42 = ae eataaP a Pay =P ‘hich isthe required locus of mid-point of AB. (a Please ty yourslt Hint: Procesd as in example 62. ‘Ans. 9x2 + Sy? = 45 Example 64, (1) Find the locus ofa point, 50 thar the join of (5, 1) and (3, 2) subtends a right dangle at the moving poi. (di) A (~a, 0) and B (a, 0) are oo feted potnts. Find the locus of the point P (x, y) which moves so that PA is perpendicular to PB. Solution. (f) Let P (x, y) be the moving point. Let A (5, 1) and B (3,2) be the given points. Now. ZAPB = 90° ++ AAPB isa right angled triangle, By Pythagoras theorem, we have PA? + PB? = ABT G-CSF + Y= + 6-37 + y-2F GFP +Q-17 40-9 + Y-27 PHBH SP Hayes 9-Gr ty tay 2 + 2y? + de ~ by + 39 = 65 202 $y + 2x39) = 65-39 202+ + 2r-3) = 26 > Pty +2 3y= 3 ‘which is the required equation of lens (i) Let P (x, y) bea moving point. Let A (a, 0) and B (2, 0) be te given points. Now. its given that: ZAPB = 90° is given that B-- 5% +Q-17 G+ 5% +0) +1 vouuyue + SAPB is a right angled triangle, By Pythagoras theorem, we have : PA? + PB? = AB? = &-CaP + 0-07 + ea + y-OF = @--ay? + -9F = Gta ty 4-0? +) =@ +a? +0 72_ Co-ordinate Goometry Made Easy Baht dart te tat tar ty aad = 20 4 2y 42d = 4a = ety teer = Payee’ Which is the required equation of locus. Example 68. (?) Find the equation of the locus ofthe point P which moves so that APAB = 10, where A (4, 7) and B (0, ~ 5) are fixed pois. (i) Find the inequation of the locus to. point such that its distance fromthe point (- 1,2) is greater than 3. Solution, (i) Let P (x.y) be the moving point. Let A (4, 7) and B (0, - 5) be the co-ordinates of fixed points. Now, itis given that area of APAB = 10. Fe Area of 4 = $ |x, 02-99) +4 03-99) +35 03-99 IT Lrg 4s +4es-y0o-n1=10 = 4 | ier-20-4 +0) = 10 = | 12e-4y-20 | = 20 = 12e-4y-20= £20 = Biter 12k ~ 49-20 = 20 0r 12r - dy - 20 = - 20 = I2r-4y = 40 or W2r-4y=0 = ar-y=10 o ar-y=0 s+ The locus of point P tx, y) is either 3x -y = 10or 3x -y = 0. Gi Let ¢r, 3) be any point onthe locus. Let A (1, 2 be the given pint Now, according tothe given condition, we have : LAP | >3 apo 9 = DP + -2F>9 w+nr+g-7>9 PHL eBay edgy 9 Pay body a5 > 9 Bayle dena od ‘whichis the required inequation ofthe given locus ‘Note: This inequation represents a region. yuuuus Cartesian System of Rectangular Co-ortinates 73 Example 66. () [0 isthe origin and Q isa variable pont on x? point of 00. (di) Find the equation ofthe locus af a point which is equidistant from the point (2. 4) and the y- Solution. (7) Let the co-ordinates of point Q be (4,3) and let © be the origin. Let P(h,&) be the mid-point of OQ. . (n 120,4.080) 2 2 4. Find the locus of the mid- 2B) 2hanty 2 Point Q (x, ),) lies on the given equation. = 4y ~ apa 4y, = Qin? = 48 = ie = 8k > Pak te The locus of point P (rk) be x? = 2y, i Let P (, y) be any point om the locus Let A 2, 4) be the given point. Now, according to given condition, point P (x, y) is equidistant of point A (2, 4) and y-axis PAs [x] = Pata? [(-: By Distance Formula} = 2 +o -a =e = Ptdede ty + 6 tya? => y-4r~8y+20=0 Which is the required equation of locus. Example 67. () Find she locus ofthe poiru of imersection of the lines x cos a+ y sin a= a and xsin ay cas a= bwhere ais variable. Gi) A point moves so that the sum of its distances from (ae, 0) and (~ ae, 0) is 2a. Prove that the ecuaon ots caste + 3 1. where b? = a? (I~ €). Solution. (() The equation of the given lines are : xeosa +ysina =a -Q) and xsina-ycosa = b -@) Let P (hf) be the point of intersection of the given lines. TA Coordinate Geometry Made Easy heosa+ksina =a 8) hsina-Keosa= A Square and Add equations 2) and (4), we get: cose + ksina)? + (ising -keosa}? = a! + 3 Poota+ Prima + 2iksina cose + IP sinha + costa -2hksina cosa = a + =? (cos' a + sin? a) + K (cos? a + sin’ a) = a + [> cos? A + sin? A = 1] = Re RAee +P = BePodsP ‘The locus of point —P(h, k) ex? +97 = a? +B, Aid Lec (ey) be any moving point Let A (ae, 0) and B (-ae, 0) be the given points. Now, according tothe given condition, we have: PA + PB = 2a e.By Distance Formula) = (Qe = ae + (yO) + fle rae + -0F = 20 = (enaeP ty = 20- [sae ay? (on squaring both sides, we have = (ene? 498 = dat 4 64 ao? 32a [eoaeP a = Pea 2ue = 4d $4 Pe? + reae-4aferaee ry = sage 4 = 4a {(rraee ey? [Dividing throughout by ~ 4a = extan faerarey (On squaring both sides again, we ba = lex tahm ot ace + = Oe +a +a =o +a + dae+y = 2U-O4+¥=80-3 [Dividing throughout by (= &)) Cartesian System of Rectangular Coordinates 75 (NBM EXERCISE FOR PRACTICE FER 1. What point on the y-axis is equidistant from (7, 6) nd (~ 3, 4)? 2. Find the point on Z-axis, whichis equidistant from he poins (3, 2) and (5, - 2). 3. Show thatthe points (0, ~ 2), (3. 1), (0, 4) and (- 3, 1) are the vertices of a square. 4. Show thatthe point (0, 6), (= $, 3) and G, 1) are the vertices of an isosceles riangle 5. Thrce vertices of a rhombus taken in onder are (2, ~ 1). G, 4) and (~ 2, 3). Fd the fourth vere. 6. Find the ratio in which the lie segment joining (2, - 3) and (S, 6) is divided by (x-axis andi) y-ais 1. ttt ods de in (6) et ne epee i 9287.9. 8. Showy sxe is ois @,- 2), (2a, ab 3, Fo cordoba af eso ees 48 2, 10 farvta vameotraw bent G22 One ets abu sho steer Ih dees af ete foe y eps 0,69. Da.) 1, Ft egestas 2) eotsa 1, er af ol no eae fn 2 a (2) 14 Pa eqn be ns ap sure hs dices eon. 3 nd 2) 6 15 Laat teepone & 0-0 rate ne otf a pererteriarea ANSWERS 1 0.15) BELO 53-2 601-202: Sexemy) | 1 ». (4) 1 -10ry 15 sq its 12 202+ Be 1 +5 eye aos WBE P28 ay Chapter 2 Straight Lines—I INTRODUCTION A straight line isthe simplest geometric curve. In this chapter we shall study the equation ofa line in 02s shown in figure (i) id) Slope ofa line is negative if 90° < © < 180° iLe., m= tan @ < Os shown in figure (i). (dv) Slope of a line is not-defined if = 90° ie.,a line is parallet t0-y-axis jue, m= tan 90°, whieh is not defined. 2.1.6, Slope of a Line Passing Through Two Given Polnts Ifa non-vertical line passes through two distinct points (x, 93). (ig. J) then the slope, m, ofthe line is given by : 78_ Co-ordinate Geometry Made Easy 2.2. PARALLEL AND PERPENDICULAR LINES ‘Theorem 1. Two non-vertical lines are parallel if and only if their slopes are equal. Proof : Let ;, [,, be two non-vertcal lines with inclinations 8, 8, and slopes my, mig respectively. ‘Let; and f; be parallel tines. y 4, + Their inclinations are equal. 4 = an, slopes are equal. Conversely Let the slopes of f, and J; are equal. x mam, le tan@, = tan, ®=8, 6, ad es Between O° and 180") 0, = 0,010, = 180° + 6, In both the cases, lines are parallel. ‘Theorem 2. Two non-vertical lines are perpendicular if and only ifthe product of their slopes is ‘minus one (~ 1) Proof : Let. fy be two non-vertical lines with inclinations @,, 8, and slopes m,, m, respectively. Let lines /, and fy are perpendicular, then, @, and Q, differ by 90°. * 0; = 0, + 90° = tan @; = tan @, + 90°) s+ the product of slopes of two lines is ~ 1. Conversely: Let my. my == 1 r = m= my = ane, =-—1— = - core, = Fitter tan6, = tan(G0° +6) or tan8; = tn, -90°) = 8 = 9 + 8) « = 6,8, = 9° or Straight Lines—1_ 79 ‘ue., 8, and @, differ by 90°. Hence, the lines are perpendicular. Remark : If the slopes of two non-vertical lines be m, and sm, thea, ( lines are parallel if and only if m, = my. (i) ines are perpendicular if and only if m, . m, = ~ 1. 22.1. Collinearity of Three Points ‘We know thatthe slopes of two parallel lines are equal. If two lines having the same slope passthrough a common point, then | the two lines will necessarily coincide, Hence, if A, B and C are three points in the xy-plane then ‘hese points will lie on a line .., these points are collinear iff slope of AB = slope of BC. 2.3. VARIOUS FORMS OF EQUATION OF A LINE 2.3.1. Equation of a Line ‘The equation ofa straight line i the linear relation between two variables. andy, which i satisfied by the co-ordinates ofeach and every point onthe line and not by those of any other point in the cartesian plane. ‘We shall see that every first degree equation likeax + by + ¢ = O would be the equation ofa certain straight line and conversely, the equation of any straight line would always be ofthe type ar + By + 2.3.2. Equation of x-axis ‘We know thatthe ordinate of each point onthe x-axis is zero. JEP (x,y) be any point on the x-axis, then, y = 0 ‘The equation of xaxisisy = 2.3.3. Equation of y-axis ‘We know thatthe abscissa ofeach point on te y-axis is 2ero, FP (x,y) be any point on the nazis, thea, x = 0. ’ 2 The equation ofy-axisisx = 0, a 2.3.4, Equation of a Line Parallel to x-axis a Let AB bea straight line parallel to thex-axis, lying ebove it ata distance b from it. “Then, the ordinate ofeach point on AB is b. a FP (x,y) be any point on AB, then y = b 2 Tae equaion of onl ie at diane from he + ‘rans, lying above the x axis is y = Similarly. the equation of a horizontal line ata distance 6 v from the x-axis, Iying below the x-axis is y = - 6 ara) 80 Co-ordinate Geometry Made Easy 2.3.5. Equation of a Line Parallel to y-axis Let AB be a straight line parallel tothe y-axis Iying on lis right hand side ata distance a from it, ‘Then, the abscissa of each point on AB is a. TEP (x, y) be any point on AB, thea x = a, ++ The equation of a vertical line ata distance 2. from the y-axis, lying on its right hand side is xa. Similarly, the equation ofa vertical Line at distancea from the y-axis, lying on its leftchand side is x = ~ a 2.4. POINT-SLOPE FORM Now, we shal find the equation of ine passing through a given point and having given slope, ‘Theorem 3. To find the equation of a straight line having given one point on the line and its slope. Proof : Let a non-vertical line passes through the fixed point A (x,, y,) and having slope m. Let Ge.) be any point onthe ine. the slope ofthe ine is 2= 2 But its slope is given to be m. =m = =) = max which isthe required equation of the Hoe. ‘This is known as Point Slope Form. Remark : Since the slope of Fine parallel wo y-axis is not defined. 1 a vertical line passes through the point, y,). Then, its equation is given by x = x. 2.5. TWO-POINT FORM ‘Theorem 4. To find the equation of a sraight lie having given two distinct points on the Tine, Proof : Let A (f, ) ad B (%, J) be two fixed points, y Let P (, 9) be any point on the line Since the line passes through the poins A, B and P. “ Slope of AB ~ Slope of AP = Yom 2 Poy aa week 4) = vo Bahama F \which is the required equation, “This is known as two point form. Straight Lines—1_ 81 ML Solved Examples I Example L. What is the slope of a line whose inclination is @) 60° ii) 90° (iy 130° fy) 30° 45" (i) 0°. Solution. Let 6 be the inclination of the lines: ‘Then, slope of the line is m = tan 8. @ When 8 = 60° Slope (m) = tan 0 = tan 60° = 3 (i) When 6=90 f Slope (m) = tan = tan 90° = The slope of line is not defined. ity When 9 = 150° Slope (m) = tan8 = fan 150% = can (180° - 30°) = tan 30° = — (>) When 8 = 30° 8 * Slope (m) = tan = tan 30 (¥) When 8 = 45° s Slope (m) = tan = ean 45° = (1 When = 0° Slope (m) = tan8 = tan 0° = Example 2 Find the slope ofthe tne passing through the pots + (@ @, 3) and (4, 9) Gi) (1, 6)and (= 4,2) ti) 3, = 2) and (= 1, 4) Gi) 2, and 5-7) (v) (0, ~ 4) and (-6, 2) Solution. (® Let the given points be A (2, 3) and B (4, 9) i [omen] (Gd) Let the given points be A (1, 6) and B (- 4, 2). 2-6 _-4 a Slope of AB= 226 = =4 ~ i. Tai 35 [: Slope (1) = | (i) Let te given points be A (3, ~ 2) and (- 1, 4). (Co-ordinate Geometry Made Easy iv) Let the given points A (- 2, 3) and B (S, ~ 7) 7-3 | -10 _-10 3--) 542 7 (©) Igt the given points A (0, - 4) and B (- 6, 2) Slope of AB = -4) 244 0 Example 3. fa lines equally inclined fo the axes, show that slope m = + 1. Solution. Let AB be the line meeting the x-axis and y-axis in points A’ and B respectively and be equally inclined with Then, ZOAB In figure () Inclination of the line AB is given by 6 = ZOAB = 45" ‘Slope of AB = 5° and ZABO = 45° + Siete == a v= cm = (180° - ZOAB) Sit in ° mmm) ga . ne Fo otis) eh Example 4. (i) ifthe slope of the line joining (2, 5) and (3, 2) is = 2. Find the value of 2 (di) Determine x so that she inclination ofthe line containing she poirus (+, ~ 3) and (2, 5) is 133. (ii) Determine x so that 2 is the stope of the ine through (2, 5) and (c, 3) 6) Fay Ye spe of te ting 8 tnd 2, i= Straight Lines—1_ 83 Solution. (7) Let the given points be A (2, $) and B (3, 4) Slope of line AB = m = -2. acs Ran = -20 An8 7 = A-S=-2 @ A2-245 @ be (i) Let the given points be A (x, ~ 3) and B (2, 5). Inclination of line AB = @ = 135° Slope of fine AB = m =tan = m= tan 135° ‘m= tan (180° ~ 45°) = ~ tan 45° = =1 = 1 sc) 543 = win 38 = -2exs8oxa842= 10 Gi) Let the given poims be A (2, 5) and B &. 3). Slope of AB = m = 2 = and [ Ste (n= 22=2| m2 naa Gv) Please try yourself. ‘Hint : Proceed as in part (1. 1 Ans. = - 5) Exampl segment ‘Solution. Let the given points are A (0, Let P be the mid-point of AB. Find the slope ofa line, which passes through the origin, and mid-point of the tine ing the potats A (0, ~ 4) and B (8, 0). ) and B 8, 0). 84 Co-ordinate Geometry Made Easy ‘The co-ordinates of mid-point P are given by CP9)-G) (4,- 2. - x [+ sem Example 6. (i) What isthe value ofy so thet the line trough (3, y) and (2, 7)is parallel tothe line through (- 1, 4) and (0, 6). (i) What is the value of y so that the line through (3, y) and (2, 7) ts perpendicular to the line through (2, 3) and (- 1, ~ 2) Solution. (2) Let m, be the slope of the line through A (3, y) and B (2, 7). my - + Slope (m) = 22= 2 rr) on [> sexe ¢ aaa] ‘And let m, be the slope of line through C (~ 1, 4) and D (0 6). Sige (n) = 2224] Roy ( Let m, be the slope of line through A (3, y) and B @, 7) m= (9-7) {For solution see part (9} ‘And etm; be the slope of line through C (2, 3) and D (- 1, - 2) - 5.5 a aK mea 373 * Slope (m) = Since, tine AB and CD are perpendicular to each exer. ‘ my. = I ‘ 3) y=7 . oa) - Sy-352-3 = Sy 0-3 435 = gy=oy= 2 5 Straight Lines—!_ 85 Example 7. () Find the slope of line perpendicular to the line joining the points (2, 3) and (12, - 9). (it) Let A (6, 4) and B (2, 12) be two given points. Find the slope ofa line perpendicular to AB. Solution. (() Let the given points be A (2, 3) and B (12, - 9) | Sloe (m) = 22= 3. x93 = B22 "70 Slope of line AB = (m,) $ Let ma be the slope of required line = (-$)tem) =-1 “ m, (i Let the given points be A (6, 4) and B (2, 12). 2-4 8 Slope of line AB = (m,) = 2=4 = 8 2-2, lope m= Fe Let ma be the slope of required line. * mm. 1 => (2)m = =! m= 3. Example 8, Show that the line joining (2, - 3) and (~ 5, 1)is (@) Parallel to the Hine joining (7. ~ 1) and (0, 3). (ii) Perpendicular to the tine joining (4, 5) and (0, ~ 2). Solution. Let m be the slope of line joining A (2, - 3) and B (- 5, 1). 4 7 (0 Let m, be the slope of line joining C (7, - 1) and D (0, 3). _ 32) set 4 0-7 ~Q) 86 Co-ordinate Geometry Made Easy From (1) and (2), we have mam, ‘The lnes AB and CD are paral, (i Let be the slope of lie joining E (4, 5) and F (0, - 2) 33. int ® \Produet of ope of lines AB and EF by 5 (-£(2) =-1 [By aig (1) and @) ‘The lines AB and EF are perpendicular. Example 9. (i) Prove thatthe line joining (6, - 4) and (~ 3, 2)is parallel to the tine, and (- 2, 5). (ii) Prove that the line passing through (- 2, 6) and (4, 8) is perpendicular to the line passing through (2, 3) and (I, 6) 0.3 Sohn Let be te slope on ning A 6, -A)and3, 2), Slope (m er 2 wo 2 i * met 4F oe Q ‘From (1) and (2), we have mam, The lines are parallel. (i) Let my be the slope of line joining A (~ 2, 6) and B (4, 8). “Slope (m) oct] an, _ 8-6 2 ™> TIE 7 a2 o Let m be the slope of line joining C @2, 3) and D (1, 6). m= 3 By using (1) and (2)) 1 Straight Lines 87 ‘Now, mom (Z]eaa-t ++ The lines are perpendicular. Example 10. Show thatthe line joining he poinss (- 2, 6) and (4, 8) is perpendicular to the line Joining (8, 12) and (4, 24). Solution, Please try yourself. Example 11. (i) Find the value of x for which the point (c, ~ 1), (2, 1) and (4, 5) are coltinear. (i) Show thatthe points (1, 1), (2. 3) and (3, 5) are coltiear. (i) Using stopes, show thatthe points (5, 1), (1, - 1) and (11, 4) are collinear. Solution. (@) Let A (x, ~ 1), BQ, 1) and C (4, 5) be the given collinear points. ‘Then, by collinearity of A, B and C We tave, Slope of AB = Slope of BC =(-) _ 5-1 ° Q-x | 4-2 = det x73 < = we = Qexaloxe2-1el, (i) Let A (1, 1), B (2, 3) amd C G, 5) be the given points, * etl? = aH Slope of AB = 3" = $ Stope(m) aaa] 5-32 and Slope of BC = 353 = 3 = 2) From (1) and (2), we have Slope of AB ~ Stope of BC. => AB and BC are parallel and have a point B in common, => A, Band C are collinear, (di) Please try yourself. ‘Example 12. Find the values of x if he polms A (2x, 2x), B (3, 2c + 1) and C (I, ) are collinear, Solution. Let the given points are A (2x, 22), B (3, 2x + 1) and C (1, 0) If the points A, B and C are collinear. Co-ordinate Geometry Made Easy Then, Slope of AB = Sipe of BC Qesya2e | - 3-2e ~ = po 4 VTEHTS 24) - ro 120 _ 4248/2 _ A(t?) 3 ¢ @ Example 13. Wishow using Pythagoras theorem, show that (4, 4), (3, 5) and (~ 1, ~ 1) are the vertices ofa right triangle. Solution. Let A (4, 4) BG, 5) and C ( 1, ~1) ate the given points lr Sloe m= = Slope of line AB = a) 3 ‘Slope of line BC = “F -@ and Slope of line AC ° From (1) and (3), we have (Slope of AB) (Slope of AC) = (= 1) (I) = ~ L(+ For perpendicular lines my mg = ~ U) => AB is perpendicular © AC => ABC isa right angled triangle, right angled at A. Example 14. (i) Line through the points (- 2, 6) and (4, 8) is perpendicular tothe tine through the pois (, 12) and (1, 24). Find the value of. (i) Wet ceaeangte does a tine of stope- 2 mate with a veal tne. Straight Lines 89 Solution. (2) Let my be the slope ofthe lin joining the points A (- 2, 6) and B 4, 8). 8-6 | 2 an ) FR 201 2-1 ¥ Slope (m) = o ‘Let m, be the slope of the line joining the points C (8, 12) and D (x, 24), wan 2 m= BoB 2 .@) ‘Since, the lines AB and CD are perpendicular. 2 mm, = =1 12 . = a3) Le. By using (2 and 3) = S-rara8-4 (Let 8, be the acute angle made by the line of slope ‘wih the vertical line : Slope of fine = m= tn 0, 2 = ~ 2 = tao, 2 mtn 2 = wind, = 2.) Now, & = @- 90") hy -s Vercaly opposite angles} = £018; = cot (@; - 90°) = cot f- (90° - 81 =~ cot 90° -8)) = - ane, = cote, z [er By using 1 aample 15. ¢) te posh, 0, 0) and (0,8 te ate, show thas» (di) The points A (a, 0), B (0, 6) and C (x,y) are collinear. Using slopes, prove that = Solution. () Let A (1, 0), B (a, 6) and C (0, K) be the given collinear points. 90 Co-ordinate Geometry Made Easy Since, the given poins A, B and C are collinear 2 Webave Slope of AB = Slope of BC = b-0 _ ko ah Ona = = = ab = (a=W (kb) = ~ ab = ak~ ab - hk + bh > ak + bh = We [Dividing throughout by he] i Please uy yourelf Hint : Proceed as in part (i). Example 16. Without using distance formula, show that the points (~ 2, ~ 1), (4, 0), (3, 3) and (- 3, 2) are the vertices ofa parallelogram. Sato teh C2, Bie 03a D 3.2) arte sents 2 OD OF Ld ny 22223 Seat FE a TE «© [sere a 2B seer @ From (1) and (2), we have Slope of AB = Slope of DC...) aca Be.0) = AB and DC are parallel Now, slope of AD 4) Slope of BC = 6) From (4) and (5), we have Slope of AD = Slope of BC oO Straight Lines—I_ 91 ‘From (3) and (6), we conclude that ABCD is a parallelogram. ‘Example 17. If the medians from A and B of the triangle with vertices A (0, b), B (), 0) and C (4, 0) are mutually perpendicular then show that : a= +: 2b. Solution. Let A 0, b), B (0, 0) and C (a, 0} be the vertices of triangle ABC. ‘Let AD and BE be the medians from A and B respectively. oo) ‘Then, D is the mid-point of BC and E be the mid-point of AC. 04a 040) _(a 1D awe (24,249) - (2,6 : conan aoan(22.238)- (2) and coordinates of Ear (22°, 22) - (2,4) 2 2 22, 80,0) oO C(a,0) [is given that AD and BE are mutually perpendicular to each other. + Slope of AD x Slope of BE = - 1 Let m, be the slope af AD. ‘and mt be the slope of BE, on an @ ° ml 2 From (1), (2) and (3), we have : = aa = a= \% Example 18, (i) A quadrilaeral has vertices at the points (7, 3). (3, 0), (0, ~ 4) and (4, ~ 1). Using slopes, show thatthe mid-points of the sides ofthe quadrilateral form a parallelogram, (i) A quadrilateral has the vertices atthe points (- 4, 2), (2, 6), @, 5) and (9, ~ 7). Show thatthe ‘mid-points of the sides ofthis quadrilateral are the verices ofa parallelogram. 92_ Co-ordinate Geometry Made Easy Solution. (2) Let the given vertices of the quadrilateral are A (7, 3), B (3, 0), C @, ~ 4) and DG,-0. Let P, Q, R and S be the mid-points of sides AB, BC, CD a and DA respectively. 143 340 co-4 cooxtnesort are (253,229) « s a Co-ordinates of Q are ( away 3is.0 Cotas of ( and Cocondinate of are 3 2-3 Now, Slope of PQ = ——2 = () 1 and slope of RS +4 +) 2 2 From (1) and @), we mie Slope of PQ = Slope of RS -@) i Slope of QR. -Pe-t 4) 2 and Slope of PS 6) From (2) and (5), we have Slope of QR = Slope of PS 6) From equations (3) and (6), we have PORSis a parallelogram, Straight Lines—I 93 (i Please ty yours. Hint: Proceed as in pant (0. Example 19. Prove analytically that the diogonals ofa square are at right angles. Solution. Let OABC be a square having each side of length ‘a’: Let O (0,0), A (a, 0), B (a, a) and C (0, a) be the vertices ofthe square as shown inthe figure, Let m be the slope of diagonal OB. m Olea Bias) ‘And m; be the slope of diagonal AC am From (1) and (2), we have : mom == 9] oy AGoK => The diagonals OB and AC are at right angles. ‘Example 20, A tine passes through (x, ¥,) and ¢h, A. Ifthe slope ofthe line ism, show thar + -y) = mx). Solution, Let A (4, y,) and B (h, k) be the piven point, ope of tine ap = Slope (m= 22=2 - a, ‘But, it is given that, slope of line AB is m. Wehave, "hy = key = mQh=n). Example 2. () Prove analytically thatthe line segment joining the mid-points of rwo sides of triangle is paraile! tothe third side and is equal 0 one-half of ts length. Gi) A 8, 4, B (3, 0) and C (7, ~ 4) are the vertices of a aca) sriangle. Show thatthe line joining the mid-points of AB and ACis parallel to BC and is half oft. Solution, () Let A (a, b), B (c,d) and C (e,f) be the vertices of triangle, Let D and E be the mid-poins of the sides AB and AC respectively. + Tcoontntes of are (245 see cee 94 Co-ordinate Geometry Made Easy sed verdana ora (222, b+f bed Now, slope of DE = ditt ~ ra wat Sipe fe » (Z=2) ~® “rom (1) and @2), we have Slope of DE = Slope of BC DE and BC are parallel Also pee |(*¢e-24¢ 22 1 =} lene = DE= Aes) (Gd Let A G, 4), B(-3, 0) and C (7, ~ 4) are the vertices of triangle ABC. Let D and E be the midpoint ofthe sides AB and AC e respectively + Co-ordinates of point Dare : 323 440) _ C4) -( tn i 347 4-4 and cord 347 4-4) | (109) 65.9 ates of oi Bae (247,24) - (2.9) = 6,0, Straight Lines 98 o-2 2 Now, Slope of DE = 35 =- 2 wt) 2 and Slope of BC = =-F @ From (1) and (2), we have : Slope of DE ~ Slope of BC. DE is parallel to BC. (+e By using Distance Formula) Also, DE = {(5-0)' +(0-2)° = (BT = BD ..@) (GuaFecacor and Be = (0 -ayf + 4-0) {oye = (ores = SWOT = Jie = 2 ++ From (3) and (4), we have L DE = 5 BC. Example 22. ABCD is a shombus. ts diagonals AC and BD intersect at the point O and satisfy BD = 2AC. Ifthe comordinates of D and O are (1, 1) and (2, ~ 1) respectively, then find the co-ordinates ofA Dut) c Solon, Lt 9b ke oni of it 7 San dgoifs bts ote henge ‘ ane La \ (Slope of AO) x (Slope of OD) = - 1 ay ave . Now, Slope of AO = 2 +) fr and Slope of OD = From (1), (2) and (3), we have (A)eo = -2y-2a-x42 > r-2y=4 3 re G42) @ It's given that BD = 2a = op = 208 (On squaring) = (OD) = 4 (OA)? (= 1P + (- 1-17 =4[@-27 + -C I) OF +627 24[@ a +449 +07 144 =4@ tae +241) 5 = 42 + 4y*- 16r +8 +20 40 + 4)? - 1G + By + 15 <0 (> Using (4)} 4Qy + 4F + 4y*- 16 +4) + By 15 =O 4 (dy? +16 + 169) + 4)? ~ 32y - 64 + By +15 =0 16)? + 64 + Oty + dy? 32y-64 + BY + 15 =0 20)? + 40y +15 = 0 Say + +3 =0 47 + 8y+3=0 Oy +3) Qy 4 =0 Ubeguuuyugge 4 1 ‘ Now, when yer s xaaye4 * xaDyed = xe2(- J tented, he vont toi a (1.2) o (2-4 Straight Lines—I 97 Example 23. Prove analytically that the diagonals of a rhombus bisect each other at right angles. Solution. Let OABC be a rhombus having length of each side as ‘a’. Let O be the oxigin and O (0, 0), A (a, 0), B(a +B, chandC (6, ) AY be the vertices of rhombus. since, 0A = 0c coo B(s+b,0) > V(o=0} +(0- 0)? = (oF +(c-0F (By using Distaee Formula) = We = (eee (On squaring) = aaPee . 99 “eo Now, the coordinates of midpoint of OB are (2742, 0+) arb “9 and the corti of mid-pcit of AC are (22%, 26) arb e zy) + The diagonals OB and AC have the same mid-point, ope matzh] aos Also, ° and Slope of Ac = <= ++» @) [ By using (2) and @)) 4+ (lope of OB) x (Slope of AC) = (> By using (2) Since, the produet of slopes of OB and AC is ~ 1. ‘Thus, Diagonals OB and AC of a rhombus bisect each other. Example 24. () Find the equation of the straight line passing through the point (6, 3) and és : (a) Parallel to x-axis. (0) Parallel to y-axis, (i) Find she equation ofthe suraight line which passes through (2, ~ 3) and is: (a) Parallel to x-axis. (©) Parallel to y-axis. ‘Solution. (2) (a) Since the given point (6, 3) Les on the line which is parallel wo x-axis. ‘The ordinate (-e., y-co-ordinate) of each point on the line is 3. The required equation isy = 3. (®) Since the given point (6, 3) lies on the line which is parallel to y-axis. ‘The abscissa (i.e., x-c0-ondinate) of each point om the line is 6. The required equation isx = 6 Gi) @ Since the given point (2, - 3) lies on the line which is parallel to x-axis ‘The ordinate (e., y-co-ordinate) of each point on the line is - 3. ‘The required equation is y = ~ 3, ® Since the given point (2, ~ 3) lies on the line which is parallel to yraxis. 2. The abscissa ({.e., x-co-orinate) of each point on the Line is 2 ‘The required equation is x = 2. Example 25. () Find she equation of a line passing through (2, ~ 3) and inclined at an angle of 135* with the positive direction of r-aus. (it) Find the equation of the ine passing through the point (2, ~ 3) and making an angle of 120° with the postive direction of x-axis. Find she equation of straight tine which passes through the point (2, 9) and makes an angle of 45° with the (+ ve) direction of x-axis. ‘Solution. () I is given tat, inclination of line @ = 135 “ Slope of tine m = tan@ = tan 135° = tan (180° - 45°) (rs tam (180° ~ 8) = ~ tan 9} = tan 4s = 1 ‘As the line passes through the point (2, ~ 3), “The equation of line is given by = y-CD=CNG-D = yeSenx+2 > xtyti=0 (di Itis given that, inctnaton of line @ = 120° s slope of line m = tan = tan 120° (> tam (180° ~ 8) = ~tan 6] @i O-y) =me-x))" Straight Lines—1 98 a = tan (180 - 60°) = = tan 60° = ‘As the line passes through the poiat (2, - 3). 6 =xt4ax-2y + 10=0, i) Please try yourself. Ans, 4r-y +30. Example 27, () Find the equation of he line through the points (— 1, - 2) and (- 5, 2) (ii) Find the equation of the line through the points (- 1, 1) and (2, - 4). (ii) Find the equation ofthe straight line passing through the points (4, 2) and (~ 2, 8). Solution. () Let the given points be A 1, 2) and B (~5, 2), ‘The equation of line passing through the points (x, y;) and (ry, y,) is given by 100 Co-erdinate Geometry Made Easy = xHy+3=0. (ip Lette given points be A (= 1, 1) and B @2, = 4). “The equation ofthe line passing through the pont (ry, ,) and Gy) given by: o-w = Baw = o-» @+D = 30-D =-5e-5 = By-3m-Sr-5 = H+3y+2=0. (dil) Prease try yourself. Ans.x+y-6=0. Example 28, (i) Find she equation of the right bisector ofthe tine joining (2, 1) and (2, 3). (it) Find the equation of the perpendicutar bisector of the line joining the poims A (2, 3) and B 6-3). (ii) Find the equation of the right bisector of the segment joining A (J, 1) aud B (3, 3), ‘Solution. (i) Let AB be the line segment joining the points A (1, 1) and B (2, 3). Let CD be its right bisector e., CD passes through the mid-point of AB. Straight Lines—1 401 Now, Slope of AB = ( ASCD.LAB 1 = Slope of CD =~ $ ve rexprocl ‘Buin of CD which passes rough (3,2) and as sone (— 4) is gvenby: 2-9) =m @—ad] 1/53 3 -2-2(x-3) 4 29-9 =-1 (2-2 o-aa-3(s-3) = 20-9 =-1(2-3) 3 = ay-4n-ne 3 u = tyne = aty-te0 © (i Let AB be the tne segment oinng the poins A (2, 3) and B (6, - 5). Let CD be its right bisector i.¢., CD passes ‘through the mid-point of AB. i ator amin (226 4. The coordinates of mid-point of ABis (22,255) wa-9 Slope of AB 2. CD pases through (4, 1). > Slope of AB = =-2 AsCDLAB - simeorc = # (vere 102 Co-ordinate Geometry Mae Easy Soto CD wih pes cg 1a sage Eigen by 1 b-e n= 76-9 be Q-y) =m =x) = 2o+ex-4 > dy t2ax-4 = x-2y-6=0 (i Please try yourself Hint : Proceed as in part (i). Ans.x + 2y-8 =0. Example 29. (i) Find the equation ofthe line passing through (- 4, ~ 5) end perpendicular tothe line joining (2,2) and (5, 6). (i) Find the equation ofthe line passing through (- 3, 5) and perpendicular tothe line through the point (2,5) and (~ 3, 6). Solution. () Let AB be the line segment having end-points A (1, 2) and B (5,6). Slope of line AB is given by cham As Let my, be the slope of CD. mem = a1 aaa . 36.6) > (Dm =-1 ‘5) and has slope ~ 1 is given by : [es o-y) = maa) [Equation of line CD which passes through the point (~ y-E9=-1k-C = yeS~-1a44 = ytSanx-4 = r+y49=0. (i Let AB be the Tine segment having end-points A (2, 5) and B 3, 6). Slope of line AB is given by : cess) ° E36) Straight Lines—1_ 103 as cb..aB Let m be the slope of CD. momncl 1 1) ge-tom= (3)moctom Equation of line CD which passes through the point (~3, 5) and has lope 5s given by : 0-9 =5h-C3) Cr O-yy =m ea O-D=56+9 y-S esr + 15 Sx-y+20=0, Example 30. Find the equation of the straight line passing through the point (5, 7) and inclined at 45° to x-axis. fit passes through potnt P whose ordinate is ~ 7. What isthe abscissa of P. ‘Solution. It is given thatthe inclination of the line is @ = 45° gouag a ‘Slope of line (m) = tan = m= tans? = mai. the line passes through the point (S, 7). ‘The equation of line is given by : O-D=16-5 SY Y= MOAN = y-T=x-5 = x-y+2=0 (0D Let P(e, ~7) be any point lies on the line (1). = Paty = ~7in (1), we get = r-GD+2=0 = r+742=0 = E+9=08x=-9 “Abscissa of point P is - 9 Example 31. The perpendicular from the origin to a tine meets it atthe point (~ 2. 9, find the equation of the line. Solution. Let OA be the given line segment having O (0, 0) and A (- 2, 9) as he end-points. [rm mon 4 Slope of line OA = mt = 104 Co-ondinate Geomeiry Made Easy Let m, be the slope of line BC which is perpendicular to OA. % mom, - Co x x uation of line BC which passes through the point - 2, 9) and has the slope 2 is given by: le O-y) =ma=a] = 0-9-2 u-ca = 99-9 =26+2 = Sy 81 = 2k +4 = 2e-9y + 85 =0. Example 32. ifthe point (a, 0) lies onthe line containing the points (at?. at) and (at, 2ats). Prove that ty «1g = ~ 1. Solution. Let AB be the line segment having A (ar,?, 2ar,) and B (a1; 2ar) as end-points. union of eine ABs piven by aes, ~2a, - > 2m) = BEM (cag) [o-n)e B= Oe~s) => (- 2at,) O28) = Teagan) 2 = =2a;) = (x= On a8) = aie) ~ (2am) (+ ty) = 2e-2an? ty + ty) = 2a? = 2anty = 2x ar? 2 6-44) = 26+ at .@ ‘Since poinc P (a0) lis on the Tine AB. From (1), we have Straight Lines—1 105 a u = heh pl Fad he nes rose pin 2 aig engl $n 2 wi eee ‘Also, find the lines parallel to them curing the y-axis at a distance of2 units below the origin. Solution. Let and J be the lines passing through (0, 2) and making anges © and 2 wit the axis. Equation of Inclination of f, wo the x-axis: 6, Slope of f, = m, = tan 6, mtn = V3 Equation off passes chrough (0, 2) and has the slope 3 is given by le O-¥) =m oa] = 9-2 = (3-9) = yo2= fix = fix -y 42-0 ® Equation off, Inciion of wo thesis: 0; = 2% Skeet, =m = ne, =n (24) ~1an(=0) Equation off passes trough (0, 2) and has the slope - Jf Jia-9 fie > fr +y-2=0 @ Let} and be the lies parallel tol andl respectively and ata distance of 2 nits below the y-axis ‘ce. fyand ly passes through the point (0, 2). o-2= O-y) =ma@-x)) = ye? 106 Co-ordinate Geometry Made Easy Equation of f, Since, fs is parallel to the line f, a slope of |; = slope of i = slope of fy = (m) = (3 Equation of line y pases through the point (0, ~2) and has slope [3 is given by = > y-G2) = J3 (e-0 fv O-y) =m @-a)) - y+2= fir = J3x-y-2=0 @) Equation off. Sine, tis parallel 10 ly s Slope of , = Slope off B Equation of line ly passes through the poiet (0, ~ 2) and has slope ~ 3 is given by = ‘Slope of !, = my = y-CD=- 3 «0 = yeoe- Jr = Sixty 42-0. ® Example 34 (i) Show sha the three polres (3,0), (= 2, - 2) and (8, 2) are collinear. Also, find the equation of the straight line on which these points lie. (i) Show thatthe poins (1, 9), (3, ~ 2) and (- 3, 16) are collinear, Also, find the equation of the Straight line on which these points le. (ii) Prove that she points (2,3), (1, ~ 2), (, 8) are collinear. Find also the equation of line on ‘hich they le. (Go) Prove thatthe pois (3,1), (1. ~ 1) and (I, 4) are collinear. Also, find the equation of the straight line on which these points le. (0) Show thatthe points (a, 0). (0. b) and (3a, ~ 2b) are collinear. Also, find the equation of the {ine containing them. Solution. (i) Let A (3, 0), B (- 2, ~ 2) and C (8, 2) be the given points. Then, the equation of line AB is given by : Straight Lines—I 107 = 2-5y-6=0 oo ) If point C (8, 2) ies on line (1). Then, it must satisfy it s Putx = Bandy = 2in (1), we get 2(8)-5(2)-6=0 = 1610-6 = =» 0 = 0, which is true. ence, the given points li onthe same stright line, whose equation is 2 - Sy -6 = 0. G0 Let A, 4), BG, ~2) and C (3, 16) be the given points ‘Theo, the equation of line AB is given by = $o-0 = y-4=-30-D 2 yoda det3 = ae+y-7=0 a Ie point C (3, 16) lies on (1), them it must satisfy it Putz = ~3, andy = 16 in (1), we get 303 +16-7-0 -~ -9+16-7=0 = 0-= 0, which is tne Hence, the given poins Tie on the same straight line, whose equation is 3x + y~7 = 0. (dip Please wy yourself. Ams. Sx-3y-1=0 o) Please Try yourself. Ans. x- 27-3 = 0. () Let A, 0), B, 6) and C (Ba, ~ 2b) be the given points ‘Then, the equation of line AB is given by bee O-a, = y-o-( Jaa (fe Q-yp = moa) 108 Co-ordinate Geometry Made Easy > ay = bx -ab = bx + ay—ab =0 @) If point C (3a, ~ 26) lies on (1) then, it must satisfy it Putz = 3aand y = ~ 2b in(1), we get = 6 Ga) + a(-26)~ab =0 = 3ab ~ 2ab ab = 0 = 0 =0, which is true, Hence, the given points lie on the same straight line, whose equation is bx + ay - ab = 0. Example 35. (i) Given the triangle with vertices A (10. 4), B(~ 4, 9), C (- 2 ~ 1). Find the equation of the abitude through B. (ii) Find the equations of the altiaudes ofthe triangle whose vertices are A (7, ~ 1), B (- 2, 8) and ca, 2). Solution. (i) Let A (10, 4), B (— 4, 9) and C (- 2, - 1) be the given vertices of SABC. Let BD be the alide on AC through B. [: <1. 6-2-1) Slope of AC (mm) Timo * Ta = 2 Let m, be the slope of BD. sie, BD LAC ‘ memel kaw ata) Ss)! 12 > (3) manlome- 2 oF BD passes through B(=4, 9 isgiven by: Cv =p) mer=xp) o-9=- 2 Ca ar = 5-9 =-120+4) = 59-45 ~~ 126-48 . . = ie t5y +3 0. (Leta (7, 0,2, 8 and C (2) beth given vertices of SABC. Let AD, BE and CF be the dice altitudes of triangle ABC, Straight Lines—1 108 =-2 Lec, be the slope of AD. Sinee, AD LBC. . mm == = COxme-1 + mot. ". Equation of AD passes through (7, - 1) is given by > yred= den & O-y) =m@-a) = a Letelope of AC = (7m) = Let mg be the slope of BE. Since, BE LAC. lam Equation of BE passes through (~ 2,8) is given by : = 0-9 =2k-C2 o-y) = ma@-a = yrB=20 +2) = y-Bered = ary 4 12=0 ® le slope of AB = (mg = SED 2 BLL Let mg be the slope of CF. Since, CF LAB. s 1m, .mg=~1 = C1). mg == 19 mg = 1 Equation of CF passes through (1, 2) is given by : 110 Co-ordinate Geometry Made Easy = 0-2 =1@-0 = yo2ex-1 = x-yt1=0. ~@ Example 36. Find she equations of the medians of the triangle ABC whose vertices are A (2, 5), B(-4, 9) and C (- 2, = 1). ‘Solution. Let A (2, 5), B (- 4, 9) and C (- 2, ~ 1) be the given vertices of ABC. Let D, B and F be the mid-points of BC, CA and AB respectively. + Coons ot Dae (=422,2=1) ae 03.4) coontimesatee (252.222) a@.y Tem ean 2 2 ant roti of Pare (E22, 28) «C3, 2 8-49) DE34) C-2,-) os st Now, ‘AD =m, = - i or. eget AD =m = AS ott . ‘The equation of median AD is given by 1 o-5) so = so-s)ax-2 => Sy-25 =x-22-Sy+B=0 - ) 29 7 SiopotBE =m = G22 = 7 ‘The eqution of median BE is given by 2 > O-N=- GE CM [vO -y) =m@-2)) = 4-9 =-7@+4) = 4y-36 =~ 7x- 28 Tx + 4y-8 @ <7 | -8 -8 - - = Slope of CF = (my) = TG 4 The equation of median CF is given by = O-7) =8h-© 1) O-y) = m-x)) > y-7=8G@+D) => yrT=8r + 8=R-y + 15=0 GB) Straight Lines 114 ‘Example 37, (i) Find the equations of the sides of a triangle whose vertices are A (- 1, 8), BU, - Dard C(-5,- 3). (i) Find the equarions ofthe sides ofthe triangle vertices are A (- 2, 8), B (1, 2) and C (7, = U. Also, find the equetion of median through the fist vertex. Solution. () Let A (- 1,8), Bd, ~ 2) and C (-5, - 3) be the given vertices of AABC. -2-8 Latslope of side ABom= Foy cana tts-3) 4+ Bquaton of side AB is gven by : 0-9 =-2h-CD) (0-0 = ma-xpl = y-8=-20+D = yoBa-2r-2 = au+y-6=0 oe emo D32e2) | 342 = Lt slope of side BC = m, = E> = <5 = TS 2. Buaton of side BC is given by : -y) = me-x91 = yCD= bond => 9G+Dax-4 = Oy +18 =x-4291-y-22=0 ® Let slope of side CA = my = 278 = HL xl s+ Equation of side CA is given by : 2 Q-9) =me-x)) u = o-)=2e-e) = 49-8) =Na@+]) = 4y-Melie+ ll 3 Mendy + =0 8) (a Please try yourself. ‘Ans. Equation of side AB = 2x + y-4 = 0 Equation of side BC = x + 2-5 = 0 112 Co-ordinate Geometry Made Easy Equation of side AC =x + y-6 = 0 Equation of median AD = Sx + dy 22 = 0 Example 38. 4 line perpendicular tothe line segment joining the points (1, 0) and (2, 3) divides it in the ratio 1 :n, Find the equation ofthe line. Solution. Let AB be the line segment joining the points A (1, 0) and B @, 3). Let point P (x, ») divides AB in the ratio By Section Formulae, the co-ordinates of point P are given by : Ix2enxt 1x3+x0) ), (24m 3 Ten Tee)" ten Ten, P acm Fa ran Now, slope oe An =(a)= 3 ab as Slope of any line perpendicular to AB = my = ~ 3 (- ve reciprocal) Ewin ote penn eit? (2 =?) tants ghenby bryos m@-x) »(erme=3) = 1(Cimeaen) itn 1¥8 = Yt Im-9=-r-net2tn = By +n) -9 = =x +n) + +n) = (+m +3 +n) -( 1) =0 whichis te required equation, Example 39. Find the equation of the bisector of 2A of AABC, whose vertices are A (~ 2, 4) BOS, 5) and C (4, - 2). Sohution, Let A (~ 2, 4), B (5, 5) and C (4, ~ 2) be the ne given vertices of AABC. Let AD be the bisector of ZA, Then, BD: DC = AB: AC = BB wy Do ac” Bee . 4,2) Straight Lines—1 113 Now, [ap | = Y(5-aF +27 By Disance Format] = JUHI = {TFB = J50 = 57 ani [AC] = fe a-ay ean ae = {CFO = [OH = B= 6/8 From (1), we have : Bo 2 5 De 6/2 6 = BD: DC =5:6 => D divides BC in 5: 6 (imernally) By Section Formulae, we have : crest 5x4 +6x5 Sx 21813) 546 3+6 (A) - ht) Now, slope of AD ‘Equation of AD passes trough (2,4) and ha the slope - + is ven by: (eyo sea) 1 O-4=- FCO = 30-4) =-1@ +2) = By 2 = -x-2 = x+3y-10=0 * which is the required equation 144 Coordinate Geometry Made Easy Example 40, Find the equation ofa tine passing through the point (2, 1) and paraltel tothe fine Joining the points (1, 3) and (- 3, 1). ‘Solution, Let AB be the line segment joining the points A (1, 3) and B 3, 1). Slope of AB = m, = 13 Let m; be the slope of line passes through the point (2, 1). Equation of line i given by ( O-y) =me-x)1 &-1) =m, @-2) wo ‘Since, the given line and line in equation (1) are parallel. Therefore, their slopes are equal. 1 ie, memel 2 From equation (1), we have 1 = yrds 56-2 = 2y-=x-2 = Qy-2ex-2 2 x= 50. Example 41. (i) In what ratio is the line joining the points (2, 3) and (4, 1) divides the segment Joining the point (1, 2) and (4, 3). (ai) Show thatthe perpendicular draw from the point (4, 1) on the line joining (6, 5) and (2, - 1) ‘vides tim the ratio 8 : 5 internally Solution. () Let AB be the line segment joining the points A (2,3) and B (4, 1) - -i3L : Slope of AB = => ‘Equation of line AB is given by fe Q-y) = m@-x)] 0-3) =-1@-2) = yoBenx42 = x+y-5=0 = @ Let CD be the line segment joining the points C (1, 2) and D (4, 3). Let AB divides CD in the ratio k : 1 at point P (x y). By Section Formulas, we have Straight Lines 116 wort sgontr (2! 322) + Point Plies on equation (1) ett 342 kel andy aed Puts in (1), we have: 4k+1 R42 Eel eed e+ 143k 42-5 +1) 0 Tk +3-Sk-5=0 %-2=0 s25k=1 -5 =0 = = = ‘The required ratio is 1 : 1 (id) Let AB be the line segment joining the points A (6, 5) and B @, 1). Let C4, 1) be any point. Drew CD 1 AB. cw. . -5 763 4 Slope of AB (my) = I= = TE =F Let my be the slope of CD Since, CD 4 AB. * mema-d ™ ACH D Be.) - (J-nsteme} + Equation of line CD is given by : O-y) = mex) 2 o-=-F@-9 = 30-1) =-248 = ay-3=-2r+8 = + 3y-11=0 ® Let point D (x, y) divides AB in the ratio k: 1 internally. ++ By Section Formulae, we have : 2+ 6 coon pi ae [224 ‘As point D lies on AB. 4116 Co-ordinate Geometry Made Easy +6 +5 Pare a ae m6 kes (Ge) Ga in(1), we have = 4412-3 415-11 HD =0 = k+27-1k-11=0 = 10k +16 =0 16 > ws eres nick ie £ => 8: 5 (internally) ‘Example 42. Find she equation of he line passes through he point (4, - 5) and parallel to the line Joining the poinss (3, 7) and (- 2, 4. Solution, Please try yourself. ‘Hint: Proceed as in Ex. 40 Ans. 3x - 5y~37 = 0, ‘Example 43, Find the equations ofthe diagonals ofthe rectangle, the equations of whose sides are x=ax=d,y= dandy =o. Solution. Let ABCD be a rectangle. The sides of te rectangle are given by : andy y ax =ay=b ‘The vertices of the rectangle ABCD are Aa. 6). BUd’, b), C (a,b) and D (a, b”) + 7 he ‘Now, slope of AC =m, = Equation of AC is given by o-%) (at a y *) @-a)[e 0-9) =ma-2)) =D) @ =a) = WF -b)(~a) @ -0) yb ~0) = @ -B) xa -b) (-b)x-(@ -ay-ab + ab + ba’ ab =0 ('-b)x-(@-a)y +ad-ab =0 -@ guuu 2: Equation of BD is given by : Straight Lines 117 0-9 = tea) Ee 0-y) = me-x)] = (-D@-a)= 6-H G-a) = yla-a)-bla-d) = (W -b)x-d (W-b) 3 x0 -b)-y(@-a)-ab' +00 + ba-ab =0 = x0! -B)-y(@-d) + ab - 8 = 0. Example 44, The Fahrenheit temperature F and absolute temperature K satisfy a linear equation. Given that K = 273 when F = 32 ard that K = 373, when F = 212, Express K in terms of F and find the value of F, when K = 0. Solutlon. Let us consider F along x-axis and K along y-axis. ‘Since, there are two points (32, 273) and (212, 373) are two points in xy-plane or FK plane. ‘As F and K satisfy a linear equation, +The equation of the ine passing through the points (32, 273) and (212, 373) is given by : > 279 = [: 100 - 273) = 1 39 = (K- 273) = Fay F-3) 5 = (K-27) = 5 (F-32) = 9(K - 273) = 5(F-32) = 9K - 2457 = SF - 160 = 9K = SF + 2297 a) Put K = 0 in equation (1), we have SF +2297 =0 = SP = -2297 > Fe Bm 3 = Fa - 459.4 118 Co-ordinate Geometry Made Easy 2.6. INTERCEPTS OF A LINE ON THE AXES Ifa stright ine AB meets the x-axis and y-axis in A and B respectively. Then, (OA is called the imereept of the Line on x-ais or x-iter- y ‘cept. The intercept on x-axis is usually denoted by ‘a (i) OB is called the intercept of the line on y-axis or lo intercept ‘The intercept on y-axis is usually denoted by ‘b'. 4 (iy AB is called the portion ofthe line imercepted between the axes ‘Note : () The intercept on the x-axis is positive if measured to the right ofthe origin, and negative if measured to the left of the origin (i The imercept on the y-axis is positive if itis measured above the origin, and negative if measured below the crigin. (id) A horizontal line has no x-intereept and a vertical line has no y-intercep. Or i 2.7. SLOPE INTERCEPT FORM OF A LINE ‘The equation ofa line with slope m and making am intercept ‘c’ on y-axis is given by : yemete Remark : () Ifthe Tine is parallel othe x-axis, then, its slope m = 0. Tis equation becomes : yeO.rtesyee (Gi) IF the line passes through the origin, then ¢ = O and the equation becomes : ym met Oy = me, 2.8. EQUATION OF A LINE IN INTERCEPTS FORM ‘The equation of a line making intereepts ‘a’ and *b on x-axis and axis respectively is given by : ‘The above result can be memorised as x +e intereept on intercept on y-axis 2.9. EQUATION OF A LINE IN NORMAL FORM xeosa +ysina =p Straight Lines 119 Proof : Let / be a non-vertical straight line, whose perpendicular distance from the origin O is ‘OL = p and this perpendicular makes an angle o withthe positive direction of x-axis. Draw LM perpendicular on-axis. oy From the figure, (ZLAX = 180° ZOAL = 180° - 0" - 0) =9 +0 * Slope of line =m, = tan = tan 60" + a) =-cota ..() me om Now, ou ML ‘Also, “ ‘The co-ordinates of point L are (p cos a, p sin a) Let P (x, y) be any point on the line * Slope of line = Slope of PL - wea = 22Psine x= peo - _ os _ y-psing sina ~ x—peosa ~ 08 © (x - p cos a) = sin a (y - p sina) = x0030. + poe? a. = y sina =p sic? o reosa + ysina =p (os? + sina) [re cost A + sin? A = 1) vuggs xeosa +ysing =p which is the required equation of the line. Remark : (9 Ifo = 0°, then the equation cos 0 + y sino = p becomes re0s 0° + ysino* = p orx(l) +y@)=porx =p ‘whichis equation ofa line paral! to y-sxis, i ex = 90°, then the equation x 608 a + y sin = p becomes x cos 90° + y sin N° = por 2O) +70) =p o yap ‘which isthe equation ofa lne parallel x-axis. 420 Co-ordinate Geometry Mace Easy 2.10. EQUATION OF A LINE IN DISTANCE FORM OR SYMMETRIC FORM OR PARAMETRIC FORM ‘The equation of a straight line passing unrough the point (xy, »)) and making an angle © with the Positive direction of x-axis is given by : xm LI-H L, ~@ cod” sine ‘where ris the distance ofthe point (x,y) onthe line from the poit (x, J). This is known as Distance Form or Symmetric Form, From equation (1), we may waite that: 22H I, 08 8 sino = xox, =reos8:y-y, = rsind > x=a troosOiy=y, +rsind ‘Thus, the co-ordinates of any point on the line are (x, + rc0s6, yy + r sin 8) and the distance ofthis point from the fixed point (x, ») is given by : x1 +7 008 =F + (reas F + (rainy? = (Feat Orr sia 6 = oP (vos? 0+ sin* 0) = VF =Irl ‘The variable rin terms of which the co-ordinates of any point on a straight line can be expressed is called a parameter and the co-ordinates are called the parametric co-ordinates, Le., (q+ reos®, y, +7 sin8) are parametric co-ordinates and ris the parameter. y A e.9) Straight Lines 124 Example 45, (i) Find the equation ofa straight line with slope ~ 1 and intercept 2. (i) Find the equation of a tine with slope ~ I and cutting off an intercept of 4 units on negative direction of y-axis. (il) Find the equation of the straight line which makes an angle of 30° withthe positive direction of axis and cuts intercept + 5 on the y-axis. to Fide stone we spe 3 nde cut 2, Sen, () Weta te a ote wi Soe mal nec yam Here m=-tanic= + 3 Lega! yeenrebe-ed = TBA! Saye a4 - weg Which is the required equation, (Gi) We know that the equation of a line with slope m and y-intercept cis given by = yame+e Here, m=-lande=—4 yECDr+64) = yaor-4 = xtyt4=0 which is the required equation. (dif) We know that the equation of a line with slope m and y-imerceptc is given by = yamete Here, inclination @ = 30° y= Lande mo tan@ = an 30" = and 1 webeve, odes "BR > By wx 53 = x f3y+5/3 =0 which is the required equation. 122 Co-ordinate Geometry Made Easy (i) Pease try youre. Ans. -Ay-5 =O. Example 46. (i) Find the equation of a line whose slope is m andthe x-intercept sd. (Gi Find the equation ofa straight ine which cuts ofan intercept of 5 its on negative direction af y-axis and makes an angle af 120° with the positive direction of x-axis. Solution. (3) Let P(x, y) be any point oa the required lite and the line intersect the x-axis at A. Since, vioercep is d Co-ordinates of A are (d, 0). ‘Slope of required line is (mm) = = yema-d Which is the required equation of line (i) We know thatthe equation ofa line with slope m and y-intercept¢ Is given by : yamte Here, inclination @ = 120° m= tan tun 60° =~ (3 im = tan 120° = tan (180° - 60°) and ones. we have, ye (V3) r+e9) = yes Jr 32 J3n ty +5=0. Example 47 i) Find she equations of the lines for which ian @ = 1, where @is the inclination of the line, and ? to pimaemrte- 2 ny nine (Find he equation ofa straig line which makes an anal ofan"! J with the aris and cuts offan intercept of - ¥ vith he ys. Solution. (F(a) We know that the equation ofa line with slope m and y-inercept cis given by = yamte, Straight Lines 123 Here, tne = + 2 We have, = = x-2y-3=0. whos slope is mand the x-interoap is ds given by : yamo-d Here, 2 Wehave, = (Gi We know thatthe equation ofa ine with slope m and yintercept cis given by : yemete Here, inclination o=tan! fF m=tnd tan (tnt 3) = JT and 2. wenave, = iy =2x-392x- fy -3-0 Which is the required equation 124 Co-prdinate Geometry Made Fasy Example 48. Find the equation of a straight line whase y-intercep is ~ 3 and which is: () paraliel ro the tne joining the poins (~ 2, 3) and (4, ~ 5) (di) perpendicular to the line Jotning the pois (0, - 5) and (- 1, 3). Solution. () We kaow that, the equation of line with slope m and y-intercept cis given by : yemere = @) Here, ca-3. [kis given thay, the required tine is parallel othe line AB joining the poins A (~2, 3) and B (4, ~5). Slope of required line i (m) = Slope of AB . “5-3 _ - - "aE 42 we have, 4 =(-4)x+09 ya (-$)xte 4 = oo bx -33y = 4-9 ye-$ a = drt ay 49e0 ‘Which isthe equation of required line. ‘Gi Wis given that, the required line is perpendicular to the Line AB joining the points A (0, ~ 5) and BC). ‘Slope of line AB = Stopeafriedneis m= = ) = £ [eve reiproa] From (), we have : y= (ete d= tea-ater- ‘which i the equation of required line. Example 49. (i) Find she equation of the bisectors of the angle between the co-ordinate axes. (ii) The perpendicular from the origin tothe line y = max + c meets it atthe point (~1, 2). Find the values of m and c. Solution. (9) Let AB and CD be the straight lines biseeting the co-ordinate axes as shown in the figure. Straight Lins 125 For live AB, inclination a= 45° ‘ ‘Slope m = tan m= tan 4s? = 1 and c-0 ‘equation of line AB is given by : yamete > y= (x +0 = yer = x-ys0 (0) For line CD, inclination 0 = 135° Slopem = tan = tam 135° tan (180° ~ 45°) = - tan 45° and c=0 Equation of line CD is given by yemre = YeCDrtOays-rartyn0 = @ (Gi Let the equation of given line AB is yemte -) Now, the perpendicular from the origin O (0,0) meets the line AB at point P(- 1, 2). Slope of perpendicular OP ism, = 2-9 = 2 2-2 Let slope of AB is m. Since, OP. AB = * mal 2 2. From (1), we have yatste Point P (~ 1, 2) lies on AB] > 2-tente 1 3 = athe =i pre eng 126 Co-ontinate Geometry Mae Easy ‘Example $0, (i) Prove that she lines x + 2y ~ and 2x + dy + 5 = Oare parallel, (i) Show that the lines 27: ~ 18) + 25 = Oand 2x + 3y + 7 = Oare perpendicular to each other. Solution, (#) The equations of the given lines are : rtd 0 ot megsseo From (, wehne yerso L 9 - yectes? ‘From (2), we have ; ya -2r-5 - yeetad pd Let m, and m, be the slopes of given equations (1) and (2) respectively 1 Fon 2 6) 2 From (4), mend % 2 sic = mse, hese ae. Theer nea (otrtecpnies tte gain be: an weyed co . arenas - 7,8 ” 1e* 18 - yadae® = geet 2 > yecder ‘Let m, and m be the slopes of equations (1) and (2) respectively. From (3) ; med a -@ @ @ a @ @ @ [v yometa Straight Lines—1_127 From (4); m= -3 Since, the product ofthe slopes of two given lines is ~ 1 ‘The given lines are perpendicular each other. Example $1. () Find she equation of the line whick makes intercepts ~ 4 and 5 on the axes, (i) Find the equation of a line whose imercepts on aes are 3 and 4. (it) Find the equation of the line which makes intercepts 2 and - 3 on the x-axis and the y-axis respectively. Solution. () We know thatthe equation ofa line making intercepts and bon the x-axis and y-axis respectively is given by : Here, = Sr t4y = 20 > Sr-4y +2050, (1) We know that the equation of a line making intercepts @ and b on the x-axis and y-axis respee- tively is given by Site ab Here, a=3andb=4 “ Ask stoartys Big tisartay= 12 =~ ax +3y -12=0. (Gi Please try yourself. Ans. 3x -2y-6 = 0. Example 52. Find the equation o the straight line which passes through the point (3, 4) and has intercept on the aces () equcl in magnitude but opposite in sign. (4) such thar their suum Is 14, Solution, We know that the equation of a line making intercepts a and 6 on the x-axis and y-axis, respectively is given by : 428 Co-ordinate Geometry Made Easy 2420 Sahat ~@ Since, the intercepts made by the line on the axes are equal in magnitude but opposite in sign. a=amdb = 2-2 stox-yna w= @ Since, equation (2) passes through the point (3, 4). 2 Wehave, 3-4-as0=-1 o From (2) and (3), we have -Iaz-yt1=0. (Since, the sum ofthe intercepts made by the lines is 14 atbald ~® ‘As equation (1) passes through the point (3,4). -O = 3(4-a)+4a=a(4-a) (v By(A):@ +b = 1495 = (15-0) = Q-3a+4a= Ma-a = Pia +42 =0 > (a-6)(@-7)=0 = a=6,7. ‘When a= 6=b= (14-0 =8 and when a=T=b=(4-Db=7. From (1), we have = 4c 3y= Mande +y=7, Example §3. Find the equation of a line which passes through the point (4, 5) and has intercept on the ares. Equal in magnitude bat opposite in sgn Solution. We know that the equation of a line making intercepts a and b on the x-axis and y-axis respectively is given by : Straight Lines—1_ 129 a Since, the intercepts made by the line on the axes are equal in magnitude but opposi a=aandb=-a @ ‘Also, it passes through the point (4, 5) We have, 4-Seae- From (2) and (3), we have : x-ys-lax-y+1=0. Example 54, Find the equation ofthe line passing shrough the potnt (2, 2) suc thatthe sum ofits intercepis on the axes is 9. Solution. We know that the equation of a line making intercepts a and b on the x-axis and y-axis respectively is given by : - @) 2) It is piven that, sum of its intercepts on the axes is 9. ie... +b = 9 1 Q) ‘As equation (1), passes through the point 2, 2). 2,2 2 24e = 2b + 2a = ab ° = 2@-0)+20=a9-a [+ By Q): + b= 955 = 9-4] = 18-20 +20 = 9a-a? = @-9a + 18 =0 = (@-6)(a-3)=0 = a=3,6, when,a=3 => b=@-3)d=6 whean6 = =O-6)=b=3 +The required equation of the line is y tor tye Lor Esa 130 Conontinate Geometry Made Easy = de +y =6orx+2y 2 +y-6 = Oorx +2y-6 Example $5, (A straight line passes though the point (2. 3) and the potion ofthe line inter cepted between the axes is bisected at this point, Find ihe equation of the line. (ii) A straight line passes through the point (1, 1) and the portion ofthe line intercepted between the sxcatis and y-axis is divided at this poins in ihe rario 3 : 4, Find the equation of the tine. Solution. (f) We know that the equation of a line making intercepts a and b on the.x-axis and y-axis respectively is given by = oD {Le de Hn nese the vans at A and rai ot Bas sho in he igre, Let point C (2, 3) be the mid-point of AB. We have From (I), we have = ar +2y-12=0 2) (Gi) We know that the equation of a line making intercepts @and b om the x-axis and y-axis respectively is given by : x4) = Let the line incersects the x-axis at A and y-axis at B as shown in the figure. Let point C (1, 1) be on the line AB and divides it in the 7 By Section Formulae, we have i 3O+80 gy dbo $.0 | ta saa - OH agg B49 Straight Lines 131 From (I), we have : = 4e43y-7 <0. Example 86. Find she equation ofa straight line which passes through he point (~ 3, 2) and is such ‘that the portion ofthe line intercepted benween the axes is divided by this polnt inthe ratio 3: 5. Solution, Please try yourself. Ans. 10r-9y +48 = 0. Example 57. Find the equation of the straight line which : () mokes equal intercepts on the axes and passes through the point (2, 3). 1) passes through the point (- 5, 4) and is such that the portion intercepted between the axes is divided by the point inthe ratio 1 : 2. Solution. () We know thatthe equation of a line making intercepts a and b on thex-axis andy-axis respectively is given by = ayy Zed @ ‘Since, it makes equal intercepts on the axes. ae = x+y 0) ‘Also, it passes through the point (2, 3). = 243 8) 4+ From (2) and (3), we have : xtyss = xty-5=0 4) (i) We know thatthe equation ofa line making intercepts a and b on the x-axis and y-axis respec tively is given by = @) Bix + ole 132 Co-ordinate Geometry Made Easy ‘Let the line intersects the x-axis at A and y-axis at B as shown in the figure, Let point C (- 5, 4) lies on the line AB and divides it in the ratio 1:2 By Section Formulae, we have 1.0+2.0 142 ant 1.42.0 T+2 = 0420 gy bt 3 = 2a =~ 1Sandd = 12 Is = $3 apap = 12 2 From (1), we bave x yy +k eto! = 2h yt ey a5" 12 . nr 5y @ = 8r + Sy = 609 Br - Sy + 60 = 0. ‘Example $8. () A straight line passes through the point P (a, b,) which bisects he part ofthe line Intercepted between the ates. Show that the equation of the Line is z + ¥ 2 (i) Find the equation of the line which passes through (1, 3) and whose y-inercept is 3ximes iis intercept on x-axis, Solution. (9 We know that the equation ofa ine making intercepts and b on the x-axis and y-axis respectively is given by 2 te ay ate w Tt intersects the x-axis at A (a, 0) and y-axis at B (, 6). Sinee, point P (a), b,) is the mid-point of AB. Straight Lines 183 We have, From (1), we have (i) We know that, the equation of a line making intercepts @ and on the x-axis and y-axis respectively is given by : 2,2 a5 {tis given that, y-intereept is three times the x-intercept. Leta = aandb = 30 From (1), we have a = a+y= 30 wn Q) ‘As it passes through the point (1, 3). = 3() +3=30 > 63% = a=2 Putting this value of in (2), we have tye3 = +y=6 = I ty-640. ‘Example $9. () Find the equations of the lines which cutoff intercepts om the axes whose sum and rodact are I and ~ 6 respectively. (i) the straight line * + = 1 passes through the points A (8, ~ 9) and B (12, 18). Find the values ofa and b. Solution. ()| We know thatthe equation ofa line making inerceptsa and b on thex-axisandy-axis respeotively is given by : () Letx-imtercept = a ‘and y-intercept = b 184 Co-ordinute Geometry Maile Easy I is given that at and @) 8) : Fomi.vene 42a o£ “ “ar 2y-6 of 2-396 = Br-2) #620 of 2x-3y-6-0 (Te given equations: ab ‘Ast pases trough he pain A, -9) = 8 90-=ab ®) ‘so, equation (1) pases trough the pon B (12, 18), . 2-15 P divides AB in the ratio 1 : 2 internally. By Section Formulae, we have : (uaz s20) 142 12 e4)-G wn 20) +1 (a) Bost) Co-ordinates of P are Also, Q divides AB in the ratio 2 : Co-ordinates of Q are ( et oa - (222.222) (§ 2) 33 33 ng be sn 0.0, 0 00” (22.8 Equation of line 2 ) i sinenby = y= Ze) = br = 2ay = bx - 2ay = 0 = @ 186 Co-ordinate Geometry Mace Easy Now, the equation of line joining the origin © (0,0) and Q Is given by 6-9 =[ (0) $-0 > ye 2u 2 = 2be = ay => Yx-ay=0 @) Example 61. Find the area of the triangle formed by the co-ordinate aes and the line ax + by = 20b. Solution. Let the given equation ofthe line is: atby=2ab ... (1) ‘Let X'OX and Y’OY be the co-ordinate axis, From (1), we have x HE Ow Let this equation intersets axis at A and yas at Bas shown i the igure : Then, OA = 2b and OB = 2a : Avenotagoa = 4.08x0A Example 62, The area of the iriangle formed by the co-ordinate axes anda tine is 6 5g. units and the length of the hypotenuse is 5 units. Find the equation of line. ‘Solution. Let AB be the given line. Let XOX" and YOY’ be the co-ordinate axes. = s0As, f= Soin eof OMB ~ Sut on = ean Os = buns Aneaof O48 = 6 “ rand =6 = ab = 12 Straight Lines 137 In right angled AOAB, (AB)? 0 = e+e . +2) ++ From (1) and @), we have : oe = 148 = @ Q5~ a) = 144 [ev P= @S-ay) = 25a — at = 144 = at - 25a + 144 = 0 = @ -9) (a 16) -” @ = 9,16 = anakad = a L444. whena = 3 From equation (1), we have aba = + Equation ofthe line is given by : Fat at4 = dc+3ys12 = 44 4y-12=0 whena = -3 From equation (1), we have ab=12 + d= Equation of the line is given by : xy ay = -4r-3ye-12 = art 3y+ 12-0 whena = 4 From egution (), wehave b= 12 => b= 2 ng 188 Co-ordinate Geometry Made Easy Equation ofthe line is given by : x Pat x = Bt dy = 1230+ 4y-12=0 when a = -4 From equation (1), we have ab= 2=o= 2 <4 + Equation of the line is given by W335 = ) Find the equation of the straight line on which the perpendicular from the origin is of length 3 units and this perpendicular makes an angle of 210° with the x-atis. Solution. (i) The equation of a straight line in normal form is given by xeosa +y sin Here, p= Sanda = 135° x¢05 135° + y sin 135° = 5 3 cos (180° ~ 45°) + y sin (180° ~ 45°) = 5 ’ =O a = P+ ysin gs 1 1 = + 3 #) (38) = naty= NF Les c0s (180° ~ 8) = ~ cos 6 sin (180° - 8) = sin 8] = opt SY2 =0. (i) The equation ofa straight line in normal form is given by : xoosa +ysina =p ~@ Here, p= Sand slope = -1 Since, slopem = tan8 = -1 Straight Lines—1 139 tan (90" +a) == 1 [e wan4se = 1) = tan (90° + a) = ~tan 45° Le tan (80° - 6) = - tan 0} tan (180° - 45°) or tan (360° - 45°) = tan G0" +a) = tan 138 of tan 315* = Sor +a= 138" or 1 +a=3I5" = a= 138" -90° or a= 315° 90° = a=45° or a= 225°. From (1), we have ous 45° + ysin45* = 5 or xe0s225" + ysin225* = 5 1 ‘i - = £05 (180" + 45) + y sin (80° + 45) = 5 Jo(p)s xen o67 erm oe 0s (180° + 8) = ~ cos @ and sin (180" + 8) =~ sin] = or -xens45*~ysinds* = 5 = aty=S\Z 0 or --E-teews . ve y = r+y-5{2 =0 or -x-y= 53 or xty 45/2 =0 (Gil) The equation of a straight line in normal form is given by = xeosatysina =p w Here, p= 2anda = 240° oe > 00s 240° + y sin 240° = 2 * * = €05 (180° + 60°) + ysin (180° + 60° r = = 605 60° ~ sin 60° = 2 [> cos (180° + 6) = ~€0s 0 and sin (180° + 8) = - sin 8} = -1- fy 4 - x4 fly +400. (i Please try youre. Ans. (3x ty +6=0. Straight Lines—I 144 = 2p = 43 [+ Area of AOAB = 54/3 Given] B = apt = 162 => pr=8l =» pat = p=3 le pis +¥¢] + Putting the values in (1), we have 005 60° + y sin 60° = 9 - Bf) = x {By = eax+ Jy -18=0. Example 66. [fp be the length of perpendicular from the origin an the line whose intercept on she axes are ‘a’ and ’b', Show that boas Poa Solution. We know thar the equation of line having intercepts @ and b on the x-axis and yeaxis respectively is given by : xy a*5 ‘Since, p is perpendicolar from the origin onthe line. Its equation should be in the normal form i.e. xcora +ysing =p «) Foosa | ysinge = mo, e ‘Comparing equations (1) and (2), we have as i ° aod se i ” cot A + sin? A= 1 ‘Straight Lines—1_ 143 = 2i2 = =r t where : ris the distance between the points (x, y) and (~ 2, 3). (ii) Equation ofa line passing through any point (x, y,) and has an inclination © with the x-axis is ssiven by a Here, > @ 2 x-D=y-3exr-yt1=0 = @) Let this line (2) meets the given equation of line x + y +1 = O at point P, such that AP = r. From @), we have : x-2s vz = x= 2+ anly=3+ vz dinates of point P are 2+ —fe,3+ He]. ecoasimrsranra (2 F520) Sie, in Ps one x+y +1 = 0 = +1=0 2+ haat vz +640 ‘ale le “444 Co-ordinate Geometry Made Eacy = reo 3VE Length of AP = |r| = 37 = ‘The length of intercept = 3/2 Example 6. The slope ofa sraigh tine through A (3, 2)4s 3. Find the co-ordinates ofthe pins on the line that are Suits away from A Solution. The slope of the given ines : 3 = m=ane= 5 a sin@ _ 3 _ sind _ cos = ae = 5 a BO 0 a eonscamn = sinO = 3k and cos = 4k = sin? @ + cos? = 1 = 30 + (4? = 9 + 16» 19288 = Lao P= = k 1 ty 3 4 sind =F andcort= = 4 Co-ordinates of point A re @, 2) and r= 5 ’ 22M IK ep cae” sing FoR pag IK ad cose ing = ee, +reosOandy =y, + rsind + ‘24 = 3) = rea (9 (28) nay 24149 (+3) - re3idmdy=243 = ge 7-Landy =5,-1. inate of required points are (7, §) and (~ 1, ~ 1). Straight Lines 145 Example 70, Ifthe sialgh ine trough the point A (3, 4) ane makes an angle © with axis and ‘meets the line 12e + Sy + 10 = Oat B. Find the length of AB. Solution. The equation of a straight line passes through the point (ry) and havi (0 with the x-axis, is given by an inclination -() Here - w Q) ‘where : ris the distance between any point P (x, y) on the ine and given point A (3. 4). + From (2), we have a-30 SS randy-4= f 8 r + Branyase 3 2 my 2 4. The co-ordinates of point B are: eB naed) Since, pont B lis onthe line 12x +5 +10 = 0 oul.) +3(é+4) +120 = 36 + 6f3r +204 2 r+ 10-0 = 66+ (65+8) r= (28s) 2 146 Co-ordinate Geometry Mace Ens a i DYs+s 132 BYs+3 Example 71. Find the equation of the straight line which passes through tite point (2, 9) and raking an angle of 45° with x-axis, Also, ind the ports on te line which one al the distance of 2 tits from (2.9). Solution, The equation of lie which passes through (x,y) having an inclination 6 is given by: Tength | BP | = 22m 2 Ion cos 8 sin @ ~ Here, 9) = @, 9) and @ = 45° > cos 45° sin 45° = ne, 0 = YE 0-2 = 29-9 = re2=y-9 = x-y+7=0 ~® ‘which isthe requited equation. From equation (2), we have Is given that - xa24 JE and yoos fF 1. The co-ordinates of the required point are (2+ 3.9 + 3). Straight Lines—1 147 Example 72. The line joining the two points A (2, 0) and B (3, 2) is rotated about A in anticlock- wise direction through an angle of 15°. Find the equation of the line in the new position. If B goes to Cin the new position, what willbe the co-ordinates of point C. Solution, Let slope of the line joining the points A (2, 0) and B (3, 1) is given by = y = = 4s° Inclination of line AB = 45° = Inclination of line AC = 45° + 15° oor Equation of a straight line passes through any point (x, y) and hiving an inclination 6 with the x-axis is given by : «Due, cos ‘sin Here, AQ, 0) and @ = 60° = ~Q) 2 4 * aBer= f(l-o +G-2) = Jisi = 2 ace 3 Purr = {2 im equation (1), we have m2 y Trae 2 3 “ a-a— LZ andy = ” xeat 148 Co-ordinate Geometry Madde Easy Example 73. If (2, 3) and B (5, 7) are 1wo pons on a line AB. Find the co-ordinates of the poimis which are at a disiance of 10 uniis from the miid-poinis of AB. Solution, The slope of the line joining the points A (2, 3) and B (5, 7) is given by : bist meh 273 “Eom 4 = mm tand = $ - snd 4 cos” 3 sin _ cos ~ SRE 6 const) = sind = Rand cos = 3k Now, sin? 0 + cos?@ = 1 = (40? + GRP =1 > 16P +9 = 1 25P = = Petok 4 3 sin = + 4 andcoso = + 2. 7 Also, -(3:3) ‘The equation of ine pastes through he point (2,5) and having a ntinaon Bs gven by: G3] cord ~ sin ard 5, [se 143 4 $+ 3 (0) andy = 5 + $10) Straight Lines 149 (56) m1 -620 Example 74, () Find the direction in which a straight ine must be drawn through the point (4, 2)30 that its poi of incersection with the line x + y = 4 may be ata disiance of 3 unis fromthe point (Gi) Find the direction in wihich a straight tine must be drawn through the point (2, 2) s0 that its point iE of sersection withthe line x + y = 4 ay be at a ditance of © wns from hs pont, * Solution. ( Equation of any line passes through the point (r, y,) and having an inclination of 8 ‘with the x-axis is given by : en nL, cos® in -a Here, G9) = 1,2) and r= 3. = . xe yo? coed” sind “@ = x+1=3corOandy-2=34ia8 = x= 3cos0- Landy =2+3sin€ +The co-ordinates of a point lies on the line at a distance of 3 units from the point (~ 1, 2) are (1 + 3.0088, 2 +3 sin). ‘As this pot lies on he line x+y =4 -1+3cos6+2+3sin6 <4 3e0t0 + 3sing =3 cos + sind = 1 [Squaring both sides) (040 + sind)? = (1? . F@+2sin@cos@ =1 [- cos’ A + sin’ A = 1 and 2sin Acos A = sin2.a) 1+ sin20=1 sin20 = 0 = sino? WH =0-0=0°, ‘The require line is parallel to x-axis. -buuuousy gq : 150. Co-ordinate Geometry Made Easy il Please «ry yourself. Hint : Proceed as in part (i. ‘Ans, (15* oF 75°). Example 75. i) Find the distance ofthe point (2, 3) from the line 2x - dy + 9 = Omeasured along the line xy +1 = 0. (d) Find the distance of the ine x + Ty + 5 = 0 from the (1, 2) along the line 2 -y = 0. Solution. (i) The equations of the piven lines are 2-329 60 a and x-y+1=0 - @ Slope fine neato (2) ven meal sorytleeyerst | cmmanveernarte cart - nm tand 1 = nase > On as" makes an angle 8 = 45° wit the positive direction of x-axis. Equation of any line passing through the point 2, 3) and making an angle of 45° withthe axis is given by : z-2 | yr3 L, Arh he, cos ~ sind con” sin : go? _ yo, tO OF = x-2e© andy-3 = 4 FEO = Fede Toamtyad4 {As this poin lies onthe ine 2x = 3y + 9 = 0 = 22+ F)-af-z)+o-0 a - e920 = ae “ fect “The required distance is 4/7 Straight Lines. 151 (if Please try yourself. 2 1 tan 8=2— sind =~ ,cos6 Hint : Proceed as in part (). [ ws 3 Ans. = 25 18 IMB EXERCISE FOR PRACTICE MEW Show that the ine through the points (0, 0) and (4, ~ 3) is perpendicular to the tine trough dhe points 4,2) and © 1,2) 2, Find the value of x for which the points (x, ~ 1, 2,1) and (6,5) are coliear. 3. AS, ~ 3), B (8, 2) and C (0, 0) are the vertices of a triangle. Show that the median from A is ‘perpendicular to the side BC. 4. A quadrilateral has vertices (4, 1), (1, 7), (6, 0) and (~ 1, ~ 9). Sbow thatthe mid-poins of the sides ‘of this quadrilateral form a parallelogram. ‘5 Find the equation ofthe straight ine passing through (3, ~ 5) and parallel to the line joining (1,2) and 3a. 46 The vertices ofa triangle are A (10, 4), B(- 4, 9) and C (- 2, ~ 1). Find te equation ofthe alte ‘through A. 7. Show thatthe points (1,4) (3, ~ 2) and (- 3, 16) are collinear and find the equation of the line passing through these point. ‘8, Find the equations ofthe sides and diagonals of the quadrilateral whose vertices ae (- 2, 6), (1, 2 (10,4) and 7,8), 9. A Tie is suoh that its segment between the lines Sx - y + 4 = and 3x + Ay ~ 4 = 0 is biseeted at the point (1, $). Find is equation. 1D, Find the equation of the line which passes through (3, -$) and cuts off intercepts on the axes which are ‘equal in magnitude but opposite in sign. 11 Fd the equation ofthe line which passes through (4, 1) and is such that the portion of the tne imercepted between the axes is divided by 12, Write the equations of the lites satsying the given conditions (© lelisation @ = 90° and distance from the origin i 5. (i) Inclination = 120° and distance from the origin is 4 1, Find the distance ofthe line 4x ~ y = 0 from the point (4, 1) measure along the line making an angle of 135° withthe positive direction of y-axis. 14, Fld the distance of the point (2,5) from the line 3x + y + 4 = O measured parallel oa line having 3 slope Z- 3 1S, Slope of sraght line through the point A (3, 2) is =. Find the coordinates ofthe points on the line that are $ units away from A. ANSWERS Bernd Sxtdy47—0 Gx-Syt+l0—0 7% 3ety-7=0 8 dr + 3y-10= 0: 2x-9y + 16 =O: dr + 39-52 =O Chapter 3 Straight Lines—Il 3.1, INTRODUCTION In previous classes, we have already studied the equation ofthe form Ax + By + C = 0, where : A, B and C are real numbers and atleast one of A and B is noa-zer0. Such equations are called General Equations of the first degree in the variables x and y. In this chapter, we shall learn the reduction of General equation ofa line in various forms, angle berweea two lines, intersection of lines, and dis- tance of a point from the line. 3.2. GENERAL EQUATION OF A LINE ‘An equation of the form Ax + By + C = 0, where : A and B are not both zero simultaneously, is called Linear Equation or a General Equation of first degree in x and y. Graph of the equation Ax + By + C = Os always a straight line. [Note + () Equation of every line Le, horizontal, vertical or oblique can be put ine form Ax + By + C= 0 For Horizon line : A = 0 For Vertical line: B= 0 For Oblique Line: A#0, B20. 3.3. REDUCTION OF GENERAL FORM TO STANDARD FORM ‘The general equation of a line can be reduced to various forms of the equation of a line = Let Ax + By + C = Obe the general equation of a straight line. ‘A.and B are not both zero. (@ Reduction to ‘slope-intercept’ form i ‘The given general equation of the line is: Ar+By+C=0 tthe form y = mx + €. - B68) 0 Comparing this with : 153, 154 Co-ordinate Geometry Made Easy yemte @ Slope of line (1) is (m) = (- 4) B . na - Soetbiien of = Co-efficient of y ant -inwncept = - Constant tr * Co-efficiem of y Equation (1) represents the equation of a ine in slope-ntereep frm (io Reteton wnecep’ Form 10 te form E42 = 1 ‘he given general equation ofthe line i: = {on dividing both sides by - C] = = 1) Comparing this with: X42 = @ = © je, - Wonstant erm = imereepe(@) = — 2 es Coefficient of x c Constant term srimtereept (b) = = ben — Co-efficient of y IFC = 0, then from the given equation, we get, Ax + By = 0 ‘which is a line passing through the origin and hence it has zero intercepts on the axes. ‘Equation (1) represents the equation of a line in Intercept Form. (if) Reduction to ‘Normal' Form : i.e, wo the form x cos 0 + y sina =p. ‘The given general equation of the line Le, Ar+By+C=0 wo {On dividing throughout by J Straight Lines—IT_ 185 A 8 c = aap aap” At+BE ° = ote ee a oo @ A+ AP + At +B Comparing tis wit xeosa+ysina =p eo Ay B where : cosa =~ sing =~ andp = Ae AB ros Equation (2) represents the equation of a line in Normal Form, Note : () To reduce the general form to normal form, transpose the constant term fo R.H.S. ang make RAS. positive by changing the siga throughout if necessary. Then. divide both sides by (Co-etcieot of =} (Co-eficen of 9 3.4, ANGLE BETWEEN TWO LINES ‘Theorem 3.4.1. To find the angle between to non-rerticat and non- perpendicular lines: Proof : Let |; and fy be two non-vertial and non-perpendicular lines with slopes m and my respectively. Let 8, and 8, be the inclinations of these lines, a my = tan ani mg = tan ‘There are rwo angles @ and (n - 8) between the lines, and ty as shown in the figure, Exterior angle : @; = 6 +6 = = 0:-6, [rs Toa angle, exterior angle is equal w the sum of interior ‘opposite anges) = tand = tan @; -8)) - tan, - und, 00 TF tan8, un 6, 2 tan@ = 727 T+ mm, ‘Also, tan (n ~ 8) = -tan@ = am = ~@ Tem ms 186 Co-ordinate Geometry Made Easy *. In Particular, If c.is the angle berween the given lines J, andl, then either a = Gora =x~9. Case 1. If is acute, then, tan ois positive, so thatthe acute angle between the lines is given by : wana = | =A 8) +m my Case 2. If a is obtuse, then. tan c. is negative, so that the obtuse angle between the lines is given by : tang = -| 2-74 @ Tm Tins te agro aa ae beeen heist desman semaio| =, siesta a | 2), rer, i te anv ons mes ide ‘acute angle between them, 3.5. CONDITION OF PARALLELISM OF LINES If two lines of slopes m, and mare parallel, then the angle 8 between them tan8 = tan0* = 0 of 0° m= m, = =0=>m-m,=0 T+ mm; ™ = m= my two Tines are parallel, then, ther slopes are equal 3.6. CONDITION OF PERPENDICULARITY OF TWO LINES If two lines of slopes m, and m, are perpendicular, then the angle @ between them is of 90° tan @ = tan 90° = Tem ms, (+ Ifa fraction is infinite, then the denominator is 22r0] > L+mm,=0 = myn 1 ‘Thus, if two lines are perpendicular, then product of their slopes is 1 Straight Lines—II_187 Remarks : 1. Iftan@ is positive, then 9 isan aeute angle between the lines and iftan 8 is negative then angle is obtuse, but to get acute angle, we reject the negative sign 2. Hf lines are in the form A+ By +C,=0 and Ax +By + =0 ‘Then, m= ines will perpendicular itt mm, = iff itt BB; iff A\Ay =~ BB: itt AAs + BB = 0 Lines will be parallel : itt iff int 3.6.1. Lines Parallel and Perpendicular to a Given Line Line Parattel to a Given Line ‘The equation of any line parallel tothe line Ax + By + C constant, Remember : To write a line parallel to a given line, change only the constant term to @ new constant k, Keeping the other terms of x and y unchanged. 3.6.2, Line Perpendicular to a Given Line ‘The equation of any line perpendicular tothe line Ax + By + C = where : > any constant ig Ax + By +k = O where :kyany isBr- Ay +k = 0. 158 Co-ordinate Geometry Made Easy Remember : To write a line perpendicular o the given line ( Interchange the co-efficients of x and y. (i) Change the sign of one of them. (iii) Change the constant term to a new constant k. MM Solved Exarnpios I Example 1. ()Reduce the equation 2r ~ 3) ~ $ = 010 slope iercept form and find its slope and yaruercepr. (a) Reduce [3x-+y +2 = O1o:the‘slope-imercep form and hence find ts stope, inclination and intercept. (iii) Reduce the equation 3x + Sy - 7 = O10 the slope intercept form and find the intercept it makes on y-axis. (tv) Reduce Sx + 6y + 15 = 010 slope-intercept form and its slope and y-icercept. ‘Solution. (i) The given equation is 2-3y-5=0 ® = a3) = 2245 = 2,5 y 3773 oe Q) (i) The given equation is : fix ty +2=0 wo = yo-Jix-2 = ® Comparing it with y = mr + 6, we get: mn-J3 ande=-2 Slope = - 3 andy-intereept = -2 {Let ie inclination ofthe tte be 8 = m=und # tang = - JF ~~ tan6o* tan (180° - 60°) = an 120° - @= 120" Straight Lines—Il_ 159 Inclination = 120°. (ii) The given equation is : 3+ Sy-7=0 =) > Sy=-3r47 3,42 > xed @ ‘Comparing it with y = me + ¢, we get 3 2 ma Faden? 3 7 Slope tm) = ~ 5 and intercept ¢ = 7 (iv) Please try yourself. sas. Soe tn) = = 4 antyinccep ==. Example 2. (Reduce 4x - 3y ~ 12 = Oto1the ‘intercept form’ and hence fin its intercepts on the axes, Reduce the equation x + 2y + 3 = Oto the ‘intercept form’, and hence find the intercepts on the (it) Reduce 3x + 4y- 5 = O10 the “intercept form’ and hence find the intercepts on the axes. (fv) Reduce the equation 3x - 2y + 4 = 0 r0 intercepts form and find length of the segment inter- cepted between the axes. Solution. () The given equation is = 4r-3y-12=0 = 4e-3y = 12 = @ Dividing both sides by 12, we have : on @ Comparing it with = +3 = 1, we have am 3andb=-4, scinercept = 3 and y-intercept = 160. Coortinae Geometry Made Ey Tee given equton is rt343=0 © = eyed ing both sides by ~ 3, we get = Q) Comparing it with a= -3ando= 3 +. sltersept = 3 andy itereepe = 3. (li) Please try yourself. cept = © and y-intercept = Ans sitercept = 2 and yintereept = 3 ((0) The given equation is : Bray +40 a) = Bx-2y=-4 Dividing both sides by (4), we get = -@) ‘Comparing it with = I wehave a=-Sanup 3 a see «Landy = Sri ni $6 ‘Let AB be the length of the part intercepted between the axes then the end-points of this line o) a8 «2. segment are Te Term _ [E _ Wo = [Baa = [RE - [5 2 as, 9 VS 8 3 ms Example 3. (Reduce [3x + y~4 = 010 he ‘normel form’ and hence fn the values ofp and a. (i) Redtuce the equation x + y+ 5{2 = O00 the normal form (iit) Reduce 3x ~ y + 2 = 0 to the normal form and hence find the length of perpendicular from origin tothe given line. (i) Reduce the equations + {[3y +5010 the normal form and hence nd the wines of rand p. Solution. () The given equation is : Sie +y-4=0 o = fir tyes = Q Hore R.H.S. is postive). Dividing both sides by of(/3) +)? = Y3F1 = JF = Wehave = 3 2 and a = 30° Hence, the required equation in normal form is x¢0530° + ysin30* = 2 162_Co-ontinate Geometry Maile Easy (0) Tre given equation is xty4sfB-0 =) sf o CCanging the sign thoughout to make R.H.S. postive. 2 =) Dividing both sides by y/(- 1)" +(-1' = JT+1 = /2, Wenave ee Comparing it with x cos a + ysin & = p, we bave = xt (4 cosa sing =~ re Since sin a and cos a. are both negative, therefore, ities in the third quadrant B = cos (180° + 45°) [= cos (180° + 6) = -c0s 9] cos 225° = a= ns * cosa te = 0s 45° in 45° Al, taan- se = sin (180° + 45°) = sin 225° = a= 25° = Sando = 225° Hence, the required equation in oral arm 00s 225° + y sin 225° = 5, id) The given equation is: aroy + = Bx-y=-2 ‘Changing the sign throughout 1o make R.H.S. positive, we have -w+y=2 . = @ w Straight Lines—I_ 163 Dividing both sides by f(-3)" +)" = J9FT = J10 -tiyyt-2 Wome, SATs Ty ~ ‘hich isthe required equation in normal form. ‘Comparing it with x cos « + y sinc = p, we have : cosa=-, sine andp = 2 To TG PS TT 1 TRetnemorpenendcarom a ovine tee sp = Be. i) Please try yourself. . 5 sre = 240 andy = $n 250 yn 240° = 8 Example 4, (Reduce the equation +4 = 0 0the normal form x cas a+ ysina =, find the values of a and p. (il) Reduce 2x + 1 = Ot0 normal form and fnd arand p (itt) Reduce the lines 3x - 4y + 4 = Oand 4x - 3y + 12 = O10 the normal form and hence determine which line is nearer tthe origin. (iv) Reducex + Jy +4 = 010 the normal form and find the values ofp and a. ‘Solution. (i) The given equation is y+4e0 ‘and hence (@) (Keeping the R.H.S, Posi ‘Comparing it with x cos a + y sina = p, we have : cosa = 0, sina = -1andp = 4, Since, sin ois negative and cos cis zero, therefore, c lies in the I quadrant. sina = -1 =~ sin 90" and cos 90° and = cos (180° + 90") and 208 270° sin (180° + 90°) = sin 270° a = 270° and p = 4. Hence, the required equation in normal form is x05 270° +y sin 270° = 4. (it) The given equation is : ay+ = doy (0) 164 Co-ordinate Geometry Made Easy = -tey=t (2) Keeping the R.H.S. positive] Divicing oth sides by (3+ OF = [FFT = J3. wehave 24104 Rep B = Which is the required form. ‘Comparing it with cos a + ysin o. = p, we have : cosa = - Fp. sinee = Fy andy = 5 = sina TE Lt casa 2 Ws =-tsaeus 7 mas-toaew Since, cos a is negative and sin cis positive therefore, ot lies in I quadrant. Hence, the required equation in normal form is: volo 3 orm (it) The given equations are Sr-4y +450 wo and ax-3y +1250 on @ = Let Brody = => Bet ays ++» (3) [Keeping the R.H.S. Positive] ] Dividing both sides by (3) + (4)? = J9F16 = (25 = 5, wehave 3 3 4) which is the required form. Here, the length of perpendicular from the origin to iis py Nox, wey => ~ 4x + 3y = 12° [Keeping the R.H.S. positive] = Dividing bot sides f(=4)° +P = (TOFD = JIS = 5, wehave Straight Lines—II_165 © vic the require form, Here the length of perpediclar from the origin otis py = “2 centy ne Te oa 37-4) + 4 =O err ob. (i) ese ny gout. nase 20" p = 2 con M0® + yin 240° = 2. Enso 5. ie prpotetar emer ofa fom te fats chaos (ii) Find the equation of a line with slope 2 and the length of the perpendicular from the origin equal of Seton. (9 Her, te perpen Letabethe an wich be perpen fom he rio he Tine makes wih axe We bv, extn oe ioral form reora + ysna = 5 is 5 units and its slope is ~ 1, cosa ‘Sina = ~ cota = -1 ( slope = - 1 (Given) = cota = 1 = lang =1 = a= 45° oF 225 s+ Equation (1) becomes : = 00s 45° + y sin 45 or 00s 225° + y sin 225° = 5 «alcale Which are the required equations. (id) Here, the perpendicular distance of line from the or Let be the angle which the perpendicular from the or P= JS 10 the line makes with x-axis 168 Co-ordinate Geometry Made Easy v3) ® ‘We have, equation of line in normal form xeosa +ysina = J3 cose Co-efficient of x Slope = - 28% = 2 + Slope =- $2-efficient of x re Sn [psa cecpaencts| = = cain (mens refa (=!) +5 ‘which isthe required equation in normal form. Aliter : Iti given that, slope of the required line is 2. Let ¢ be the intercept on the y-axis. We have, equation of line in slope-intercept forma : ~Oleya mete yamte = -ttyee 2) [Keeping the RLS. postive) Dividing bom sides by {(-2) +) = YTFT = J3, weave: 2 tle = hehe k 6 | whic i the equation of tine in normal form. ‘Comparing it with: xoosa+ysina =p 2 nga © = cose = = Fe sina = Fy andp =F. s given that, the length of perpendicular from the origin is [3 . = Fer Baers. Equation (1) becomes : yewes which is the required equation of line. Straight Lines 167 Example 6. Which of the lines xy + 3 = Oandx~4y-7 Solution. The equations of the given lines are is farther from the origin. x-y43=0 oD and xn4y-7=0 @ Le reyes = -r+y ©) [Keeping the RH.S, positive] Dividing bom sides J(—1)' +()* = JTFT = J2, wehave ty ty.3 “BU AR ® Here, the length of perpendicular from the origin to it is p, = re Now, aayeT + (S) (Keeping the R.H.S. positive] Dividing both sides by (E+ (@4) = YTFI6 = /77, we have pox-tey= oh © Taye ar . Mee heh of pen foe ont = Pg Clearly, p> Ps ‘The line x - y + 3 = 0 is farther from the origin. ‘Example 7. Show that the origin is equidistant from the three straight lines. aay + 10=0, Sx- [dy + 26=0 cand Te + Dy = 50. Solatoa. The equations ofthe given lines are: ar +374 10=0 -«) Sx-12y +26 @) Te + 24y-50=0 on From (1), we have dr +4y = =4r-3y = 10 - @) [Keeping the R.H.S. positive} Diviaing bow sides by f(—4)" +3 = JTSFS = JTS = 5, wenave 168. Co-ordinate Geometry Made Easy 3 progr? () Here, the length of perpendicular from the origin to itis py = 2 From (2), we have Sr- 1p =-26 = = Sx + 12y = 26 . (6) [Keeping the R.H.S. positive] Dividing both sides by (- 5)° + 12)" = (254144 = (163 = 13, we have : = 0 Here, the length of perpendicular fom the origin wo py From (3), we nave Te + 24y = 50 «+ (8) UKeeping the R.HLS. positive} Dividing both sides by J (7)° + (24)) = (494576 = (625 = 25, we have: = ~® Here, the length of perpendicular from the origin to itis py = Clearly, Pym Py py 2. Hence, the given three straight lines are equidistant from the origin, Example 8. () Find the angle berween the lines 3x + y~7 = Oand x + 2y +9 = 0, (ii) Find she acute angle berween the lines x + y + 1 = Oand 3x + y~3 = 0. (ii) Find she acute as well as obtuse angle berween the lines y = (2~ J) x + 6andy = (2+ J3) 5-9, (iv) Find the angte benween the lines y - {3x ~5 = Oand J3y -x + 6 = 0. (0) Find the angle berween the lines x ~ yf3 ~5 = Oand Je + y~ Solution. () The equations ofthe piven lines are Br ty-7=0 a) x+2y+9=0 = @) Straight Lines—I_ 169 ‘Let my, and mi be the slopes ofthe lines (1) and (2) respectively. and = O= 45°, (id) The equations of the given lines are : @ @) Let, and my be the slopes ofthe lines (1) and 2) respectively Co-efficient of Coefficient of y m= = Landm = - JT [sure = Let be the acute angle between the given lines, 14643 |_ B= yaige Ge ota 13 | Voet = fan 6 = tan 15° = 15° (ity The equations ofthe given lines are =(2-J3)x+6 oA) (2+ /3)x-9 6 ® 170 Co-ordinate Geometry Made Easy ‘Let m, and m be the slopes of te lines (1) and (2) respectively. m=2-f3 [+ In equation ; y = mx + ¢, mis the slope of the line] a m=24 8 ‘Let be the acute angle between the given lines. wn =| =| (-V3)-2+V9) me emmy | | 1+ = V3} @+ 13) | =a =f. N3_|_|-23 ier -(ay]| eal “hs = tan0 = tan 60° = 6= 60 + The acue angle between the lines is@ = 60° and the obmuse angle berween the lines is (180° -0) iue., (180° ~ 60°) = 120°. (iv) The equations of the given lines are : y- fie-5-0 A) Vy -34+6=0 we Q) Let m, and m; be the slopes ofthe lies (1 and (2) OA 5 Covfcen of 1 ‘Cocetficiea of y Cola and m= Ba et ® be the acute angle between the given ines. : tne a54| Term, 1 2 -| 2% B Straight Lines 171 mun = Fy - Jy Hn" = = 30" (0) Please try yourself ‘ans. 9 = 90°, Example 9. (i) Find te angle between the lines: (= 0b)y = (ab + Bx + band ab +c!) y = (b= bi) x +e where a> b> 0. (W) Find the obtuse angle between the lines x ~ 2y + 3 = Oand 3x+y-1=0. (tii) Find the angle between the lines x-2 = Oandx + f3y-5=0. Solution, (j) Let the equations of the given lines are : (@~0b)y = (ab + Bx 4D ” and (e+ a) y= (ab-yz +a @ Letm, and ms be the slopes ofthe lines (1) and (2) respectively. andms (oP) Fs _ Costin] (oe) Coreffcen of 3 mam) _|_ i+mm mbes and = bla-b) (@- Bard) we “e wlale ss) 4a? | oP | |e er) [ra>b>0) 172 Co-ordinate Geometry Made Easy (Gy The equations ofthe given lines are x-243=0 Bxty-1e0 Let m, and m, be the slopes of lines (1) and (2), respectively. w @ : methel 2 Co-eiiclent of x * I" EH? Coefficient of y and means ‘Let 0 be the acute angle between the lines. tang = | ism 1 tea on (en = and = 790» un! “he acute angle between the ies is 0 = ta" 7) and ihe obtuse angle between ihe ines (180° ~ 8), = (8-0) = [r- ws (it) Te equations of the given ines ae x-2=0 xtf-s-0 .@ Since, x-2 = 0 he, 4 = 2a vert ine, +. etre the inclination ofthe inex + fy -5 = 0 Slope (m) = tan c= “5 = ana tan 30" yp = sana = tan (180° ~ 30°) {re an (180° ~ 8) = ~ tan 6} = un 150° = a= 150° ‘Let O be the acute angle between the given tines, then Straight Lines 173 a= 90 +8 £ a= 150°] = 90° +0 = 150° = 50° — 90° = 8= 6" ‘Example 10, () Find she angle berveen ihe lines Joining the points (0, 0), (2, 3) and (2, 2), (3, 5). (ii) Find the angle berween the lines joining the points (~ 1, 2). (3, - 5) and (~ 2, 3). (5, 0). Solution, (0 Let m, be the slope ofthe line joining the points (0, 0) and (2, 3). “ ot. +: Slope #222 & meted [stone al = tan (i Let be the slope of the live joining the poims (~ 1, 2) and (3, ~ 5). -$-2 -7 7 3-En 341 And, Tet m be the slope ofthe line joining the points (2,3) amd (5, 0 [ Stpe 2H ] nA m= <2 2 2° 5-2) 542 7 Le beste eu sgl tewen te es avn slopes my and . 1 tan@ = | 2% T+mmy 174 Co-ordinate Geometry Mate Easy Example 11, (The acte angle beoneen ovo lines is and slope of one of them is. Find the slope of the other. (ii) The slope ofa line is double ofthe slope of another line. If he tangent ofthe angle between thera 63, grate spe of ets Solution. (d Let @ be the acute angle between the lines having slopes m, and m;. Here, let = Straight Lines—IU_ 175 ‘estore eter = or, (dd Let slope of one of the line is m ane the slope of another tine Is 2m, Let be the acute angle berween the lines having slopes m and 2m. m-2n ane = Tem em oe (LD) eis given that, tnd = 5 = 2. From (1) and @), we have 1 *3 = a o Team *3 = wan =1+2nt ot = Qn? +3n+1=0 or 2nte3m+1=0 = (n+ DQm+)=0 o (m-Qm-)=0 1 = on mail 2 Since, the slopes of the lines are m and 2m, therefore, the required slopes are 12340 2 Example 12. (i) Find the equations ofthe lines through the point (3, 2) which make an angle of 45° with the line x ~ 2y = 3. (i) Find the equation of vo Hines making on angle of 45° withthe line 6x + 5y~ 1 = Oa passing through (2, ~ 1). (ii) Find the equations of the ovo lines which can be dravm through the point (2, 2) to make an angle of 5° with the line x + y = 2. (iv) Find the equation ofa line passing through the point (2 3) and making an angle of 45° with the Hine x + y~5 = 0. ‘Solution, () The equation of given line is: x-2y-3=0 a (D Let, be the slope ofthis fine: a. Co-efficient of. ‘Coefficient of y 176 Co-ordinate Geometry Made Easy 1Letim be te slope of the tine passing through the point (3, 2) which makes an angle of 45° with the gives tne (1) woo | ot 2 O= 48") Lm m, Le = tan 45° = | 2 [+G)eo “The cquations ofthe required lines are given by +> Equation of line i: (7-9) = mOe-xpl ~ O-D=IW-3 oF G-2=-F 6-9 = -2e8-9 or Hy-6 ann > aeay-7= or r+ 3y-9=0 (Gi) The equation ofthe given ine is: Gr + Sy-L= 0 + C) Lem, be the slope of this Line 7 Co-efficient of x [: sem ge] [Let be the slope of the line passing through the point (2, ~ 1) which makes an angle of 45* with the given line (1), = 457] Straight Lines—Il_ 177 = = = = =6n3m oy - = =U =1 or Equations ofthe required lines ere: yeGD=N@-2 0 or y 7? ~ yRI= tle of Me =e 42 = lix-y-23=0 or Hy Fe 2 or xt lly +90, i) The equation of the given line is: xty Let m; be the slope of tis ine. Oy Co-efficient of x Co-efficient of y ‘Let m, be the slope of the line passing through the point (2, 2), which makes an angle of 45° with the given line (1). . m, = 454] 178_ Coordinate Geometry Made Easy u v ‘Tous, one of the required line passing through (2, 2) is given by Y-2= 06-2 Y=») = mel = y-2=0 [s m=0) = yer ‘The another line ought to be vertical asthe slope is nat defined in that case. z+ The equation of this ine isx-2 = O=9 x = 2 [r+ it passes through the point (2. 23] Gv) Please try yourself, Ans.x + 2y-8 = O0r2x-y-1 ‘Example 13. (i)Find the equations of the two straight tines through (7, 9) and making an angle of 60" wit the tine x - {39 -2Y3 = 0 (i) Find she equation of the lines through the origin and making an angte of 60° wih the line xeyB4N5 =0. (iti) Find the equation of the ines throughs the point (3, ~ 2) and inclined at angle 60° to the line Virty =i Gs) Find the equtions ofthe tnes which passthrough (5) ad make an angle of 5° wih the Une weytt=0. Solation.() The equation ofthe given line is x-J3y-2/3 =0 <= () etm, be the slope ofthis ine a1 1 Co-efficient of x “ poche stope == C2-eikient of MO | OPE =~ Co. efficient of y Let mabe the slope of the line passing through the pint (7,9). which makes an angle of 60° with the given line (1). 1 Thm Straight Lines—UU_ 179 = ‘Thus, one of the required Tine passing through (7, 9) is given by = =-Le- -y) = meee. 0-9 == Fee O-¥) =m & ap] ~ VBy- 9B = =x 47 = xa fiv-7 +93 ‘The another line ought to be vertical as the slope is not defined in that case. The equation of this line is x— 7 ("The given equation of the line is = x27. atyf3+33 =0 od) Let m be the slope ofthis line. = Toe Stope =- Cevetficen of x me [Ff Store Seem et] Let m be the slope ofthe line passing through the origin i.e. (0, 0) which makes an angle of 60° ‘with the given line (1). fs 0 = 60°] 180 Co-ordinate Geometry Mai Easy = 3 = ~1- Jim, = -3 + Jim, = = 2Y3m, = -2 or m= arcs ‘Thus, one of the required line passing through the origin f.e., (0, 0) is given by : 9-0= Fee (ye Q-y) = map] = Ver = x-V3y =0. “The another line ought robe vertical asthe slope isnot defined in that ease, “The equation ofthis line is x = 0. (iy The equation ofthe given line is : fir ty-1=0 @ Let my be the slope ofthis line. - +: Slope = Scanoners Let be the slope ofthe line passing through the point (3, ~ 2), which makes an angle of 60° with the given line (1) Straight Lines—it_ 181 = tan 60° = ee 60") = oo Be 8 = = YB my = J3—3m or ~ f3—my =~ fF 43m = 3m, = 2Y3 or = ma V3 or "The equations of the required fines ae given by [0-99 =me=xy] W-CI= V3 @-3) of y- © 2) = 0-3) = yea Jie-3/F of y42=0 = Vix -y +2443 =0 oo y#2=0. () Please try yourself Ans.x + 3y-19 = 0; 3r-y-7=0. Example 14. (Prove thar the poins (2, ~ 1), (0, 2), (3, 3) and (S, 0) are the vertices of a parallelogram. Also find the angle Berween she dfagonas, (8) Shove tha he points (2, ~ 1), (0.2), (2, 3) and (4, 0) are the vertices ofa paralelogramt. Also, Jind the angle between the diagonals. Solution, (i) Let A (2, ~ 1), B (0,2), C (3, 3) and D (S, 0) be the given verices, ..-Mid-point of diagonal AC B cas) and mid-point of diagonal BD = aad 30.2) 182_ Co-ordinate Geometry Mae Easy Clearly, mid-poimss of diagonals AC and BD are same, :. The diagonals AC and BD biseet each other. 2 ABCD fs patton Sipe ==] Now, siope of AC im) = CY = 341 ag 2-0 2 Slope of BD (my) = F=9 = 2 Let 9 be the acute angle between the diagonals then, 2) ja aesl lis 2) _ 22 > pela als 3 5 (di) Let A @, ~ 1), B(O, 2), C @, 3) and D (4, 0) be the given vertices. + Mi€-point of diagonal AC and mid-point of diagonal BD “(35 Cleary, mid-point of diagonals AC and BD are same. “The diagonals AC and BD bisect each other ABCD is a prallelogram, = Slope =22=2, Po BoCH) 341 2-2 0 Now, slope of AC = (rm) = + AC is a vertical line, Slope of BD = (m,) ‘Let 0 be the inclination of BD with the positive direction of x tana = ZODP = (180° - 6) [> tam (180° - 8) s-(H9H ~tand = tan (180° - 6) 2 yi fl - cu -9)= (2) Auge nec dps AC an BD's ny ZAPD = 90° - ZODP = 90° = 90° — cot! 2 = tan? 2, _,_,. Example 15, (Find the equation of the line which has y-intercept 4 units and is parallel to the line 2 ~ 3y = 7. Find the point where it cus the x-axis. (@) Find the equation of a straight line which is parallel to 2x- y + 8 = Oand having y-intercept 4. Solution. (3) The given equation of line is: 2x-3y-7=0 =) Let m, be the slope of this line, ‘Since, the required line is parallel to the given line (1). Is slope m) = 2 send Also, y-interoept is 4. i, = ye {+ Equation of line in slope-intercept form : y = mx + c] = Wart 12 = 2r-3y + 12=0. (i) The given equation of line is:

You might also like